Docsity
Docsity

Prepare for your exams
Prepare for your exams

Study with the several resources on Docsity


Earn points to download
Earn points to download

Earn points by helping other students or get them with a premium plan


Guidelines and tips
Guidelines and tips

Medical Case Studies: Various Conditions and Treatments, Exams of World Politics

A series of medical case studies, covering a wide range of conditions such as appendicitis, bladder cancer, sinus inflammation, hemolytic anemia, esophageal dysfunction, graves' hyperthyroidism, and more. Each case study includes the symptoms, diagnosis, treatment, and potential complications. This document serves as a valuable resource for medical students, healthcare professionals, and lifelong learners interested in understanding various medical conditions and their management.

Typology: Exams

2023/2024

Available from 05/14/2024

A-Grade
A-Grade 🇺🇸

691 documents

1 / 80

Toggle sidebar

Related documents


Partial preview of the text

Download Medical Case Studies: Various Conditions and Treatments and more Exams World Politics in PDF only on Docsity! World Step 2 Question bank Exam Latest with Verified Answers. Where is the classic pain in diverticulitis? (Vs. appendicitis) - Correct answer Left lower quadrant (vs right lower quadrant) What vitamin is beneficial in treating children with measles? - Correct answer Vitamin A What's a common side effect of dihydropyridine CA-channel blockers (ex: amlodipine)? - Correct answer Peripheral edema via dilation of peripheral blood vessels What receptors does risperidone act on? - Correct answer Atypical antipsychotic - *DA antagonist* that also acts on *serotonin receptors* - risperidone found to ink DA levels more than other antipsychotics What receptors does aripiprazole act on? - Correct answer Partial agonist of DA *D2* receptors (increases DA) What is hydrated disease? - Correct answer Infection with *Echinococcus granulose* from contact with dogs -> can cause unilocular cystic lesion in any organs (liver, lung, muscle, bone), usually multiple lesions -> "eggshell calcification" on CT. TX: albendazole + surgical removal. How do simple liver cysts form? - Correct answer Congenital; fluid secretion by liver epithelium -> dull RUQ pain, abdominal bloating, early satiety What's acute stress disorder? - Correct answer Similar to PTSD - pt. experiences event that threatens serious injury/death, six’s present *< 1 mo.* What are the genetics behind Marfa’s? - Correct answer Autosomal dominant mutation of *fibrillin-1 gene*, up to 30% of cases are sporadic How does the lens dislocation differ between Marfa’s and homocystinuria (cystathionine synthase deficiency)? - Correct answer Marfa’s: lens dislocation upward Homocystinuria: lens dislocation downward How does Ehlers-Dandles syndrome present? - Correct answer Disorder of collagen structure: hypermobile joints, easy bruising, poor wound healing, soft/velvety/hyperplastic skin, ~organ rupture/hemorrhage What murmur is associated with infective endocarditis (ex: IV drug users)? - Correct answer *Tricuspid valve regurgitation* -> holosystolic murmur that increases with inspiration What murmur would you hear with hypertrophic cardiomyopathy? - Correct answer *Systolic ejection murmur (SEM)* that increases when patient stands from a supine position (Dec venous return to the heart, worsens dynamic gradient) Should we screen for bladder cancer? - Correct answer USPSTF recommends against screening for bladder cancer - low incidence, poor PPV of screening tests (UA & cytology, bladder tumor antigen, nuclear matrix protein) What are the most common organisms behind acute bacterial rhino sinusitis? - Correct answer Strep pneumonia (30%), noticeable H flu (30%), & Moraxella catarrhal is (10%) What are some alarming six’s of life-threatening fungal sinusitis? - Correct answer Epistaxis, turbinate destruction, palatal scars, and maxillary cyanosis Definition of chronic sinusitis, and a causal pathogen? - Correct answer Sinus inflammation *> 12 wks.*, can be caused by Staph aurous What is 1st line pharm TX for bipolar d/o? - Correct answer 1. Atypical antipsychotics (risperidone, aripiprazole, olanzapine) 2. Lithium (excreted thru kidneys! not for ARF) 3. Valerio acid *Combo TX has more rapid onset of action What is Escher? - Correct answer Firm necrotic tissue classically formed on exposed tissue following a burn -> when occurring circumferentially, it can restrict outward expansion of compartment -> compartment syndrome What're the 5 P's of compartment syndrome? - Correct answer Pain (earliest sign, out of proportion to injury), Paresthesia’s (burning or tingling in distribution of affected nerve), Pallor of overlying skin, Uselessness (late finding), & Paralysis (late finding). What does *central cord syndrome* classically look like? - Correct answer Hyperextension injury in an elderly pt. with degenerative changes in cervical spine -> selective damage to central part of anterior spinal cord -> weakness that's more pronounced in upper than lower extremities (motor fibers serving arms are nearer to center of corticospinal tract) What does *posterior cord syndrome* look like? - Correct answer bilateral loss of vibratory & proprioceptive sensation What does *anterior cord syndrome* look like? - Correct answer bilateral spastic motor paresis distal to lesion, usually 2/2 occlusion of vertebral artery Signs of TTP-HUS? - Correct answer Unexplained *hemolytic anemia* and *thrombocytopenia* in a pt. w/*renal failure* and *neuron six’s*, fragmented RBCs What are some anti-cytokine agents used in rheumatoid arthritis? - Correct answer Infliximab, etanercept, adalimumab - rare side effects include predisposition to infection, cytopenia, and lymphoma How to reduce adverse effects of methotrexate (anti-metabolite)? - Correct answer using concomitant folic acid How is HbA1c formed? - Correct answer Non-enzymatic gyration of hemoglobin; its level is dependent on blood glucose level - generally reflects age blood glucose of last 3 months (life span of RBCs) What is sick thyroid syndrome? - Correct answer Pt. w/acute severe illness may have abnormal thyroid function tests - thought to be due to caloric deprivation and increase in cytokine levels (IL-1, IL-6); most commonly just *low T3 syndrome* (Dec peripheral conversion of T4 to T3), and *transient ink in TSH* during recovery; severity of illness and overall prognosis roughly proportional to severity of TFT abnormalities What happens to TFTs in primary clinical hypothyroidism? - Correct answer Dec total/free T4, ink TSH levels; serum T3 remains normal until late stages of hypothyroidism What happens to TFTs in subclinical primary hypothyroidism? - Correct answer Normal T3 & T4, ink TSH levels What happens to TFTs in central hypothyroidism? - Correct answer Dec T4, normal TSH & T3 levels (centrally not responding to low T4) How do you determine the presence and quantity of feta-maternal hemorrhage? - Correct answer Presence of hemorrhage = *rosette test* = qualitative. Quantity of hemorrhage (used with (+) rosette test) = *Kleihauer-Betke stain* or *fetal red cell stain* using flow cytometer. -> Then correct the dose of anti-D I When should you administer the standard dose of anti-D I in pregnancy? - Correct answer at *28 wks.* of an uncomplicated pregnancy (risk of all immunization before that is very low) Why are initial menstrual cycles in pubertal females often irregular/an ovulatory? - Correct answer Immaturity of the developing hypothalamic-pituitary-gonadal axis -> doesn't produce adequate quantities/proportions of LH & FSH required for ovulation -> endometrium builds up (estrogen) but cue to slough it is lacking (no progesterone b/c no ovulation) -> estrogen breakthrough bleeding What happens with XS LH secretion? - Correct answer Polycystic ovary syndrome (PCOS) - also, androgen XS (leading to hirsutism) How does your liver maintain blood glucose levels during fasting? - Correct answer First 12 hrs. glycogen reserves drop, gluconeogenesis is important. After 24 hrs. gluconeogenesis becomes sole source of glucose. What're the main substrates for gluconeogenesis? - Correct answer 1. AA's (from muscle protein - mainly *alanine*), 2. Lactate (anaerobic glycolysis), 3. Glycerol-3- phosphate (from TAG in fat) What do subdural hematomas (tearing of bridging veins) look like on non-contrast head CT? - Correct answer White crescent, +/- midline shift from mass effect How do lacunar strokes occur, and where? - Correct answer Small vessel halitosis (HTN, diabetes are risk factors) -> occlusion of small penetrating arteries, often in the *internal capsule* -> 25% of ischemic strokes -> *pure motor dysfxn* How does Parkinson's tremor differ from essential tremor? - Correct answer Parkinson's tremor: occurs during rest, improves w/activity - often the first sign of Parkinson's. Essential tremor does the opposite (occurs w/activity). What's trihexyphenidyl? - Correct answer Anticholinergic agent, typically used in younger pts. Where tremor is the predominant six of Parkinson's What are some treatments for essential tremor? - Correct answer 1st line: propranolol +/- pyrimidine (AED). 2nd line: BDZ or clozapine. What CV effects does furosemide have? - Correct answer Decreases cardiac preload -> Dec palm capillary P -> relieves palm edema in heart failure. Vasodilation -> further Dec cardiac preload. What do you see in peripheral blood smear for pts.? with *G6PD deficiency*? - Correct answer Bite cells and Heinz bodies What antibodies are involved in autoimmune hemolysis? - Correct answer Anti-RBC membrane Iggy antibodies -> positive *direct Coombs test*, peripheral blood smear with *spherocytosis* (RBCs removed from circulation by spleen) What's the typical presentation of nasopharyngeal carcinoma? - Correct answer usually asymptomatic until ds is advanced. Six’s: recurrent otitis media, recurrent epistaxis, nasal obstruction. What risk factors are associated with undifferentiated nasopharyngeal carcinoma? - Correct answer *EBV* - titer levels may be used to track the progress of therapy for NPC. Also, smoking and chronic nitrosamine consumption (salted fish). Chronic alcohol use is a risk factor for what diseases? - Correct answer Alcoholism, alcoholic cirrhosis, aspiration PNA, Wernicke-Korsakoff syndrome, depression, & liver cancers. What do the diff stages of syphilis look like? - Correct answer Painless chancre -> disseminated skin lesions, condylomata late, & fever -> ascending otitis, tabs dorsal is, psychosis, & tumors (gammas) of skin/bone/liver What cancer is H. pylori gastritis associated with? - Correct answer Gut-associated lymphoid tissue (GALT) lymphoma What's the classic presentation of intraductal papilloma? - Correct answer Intermittent blood discharge from one nipple, premenopausal women, small size (may not detect on U/S), soft consistency, use situated beneath areola How does fibro adenoma usually present? - Correct answer Solitary breast lesion, painless, firm, mobile lump, ~2cm, use in women ages 15-25, benign, no change with menstrual cycle How does ductal carcinoma in situ usually present? - Correct answer Postmenopausal women, use discovered as incidental finding on mammography, six’s include nipple discharge & breast mass, his to dx (ductal epic ban, no penetration of BM) How does neuroblastoma usually present? - Correct answer Most common extra cranial solid tumor of childhood, ~2yo at dx, from *neural crest cells*, most common site is abdomen (adrenal glands or retroperitoneal ganglia), calcifications & hemorrhages on imaging, up to 70% have metastasis (long bones/skull/bone marrow/liver/lymph nodes/skin) at presentation, elevated serum/urine catecholamine’s & metabolites (*HVA & VMA*) Which tumor arises from metaphors? - Correct answer Wilma' tumor (renal parenchyma) - use presents in early childhood (2-3 you) as asymptomatic bad mass and hematuria What does mesonephros give rise to? - Correct answer Seminal vesicles, epididymis, ejaculatory ducts, & ducts deferens What does paramesonephron give rise to? - Correct answer Fallopian tubes, uterus, & part of the vagina What is Morton neuroma? - Correct answer mechanically induced neuropathic degeneration, commonly occurring in runners. Six’s: numbness, burning of toes, aching, burning in distal forefoot, worsened by walking on hard surfaces. Pecks: pain btw 3rd & 4th toes on plantar surface, clicking sensation (*Mulder sign*) when palpating this space & squeezing metatarsal joints. TX: metatarsal support *bilaterally* via inserts, or surgery if this fails. How does duodenal atresia present? - Correct answer Bilious vomiting that's not associated with the first feeding. Asoka w/*annular pancreas*. On radiology, *double bubble sign*. How does arterial occlusion (embolus, thrombosis, or trauma) present? - Correct answer Pain, *diminished pulses*, pallor, coolness to touch, neuron deficits, muscle dysfxn in affected extremity How is the presentation of arterial thrombosis different from that of arterial embolism? - Correct answer Thrombosis = slow, progressive narrowing of vascular lumen, so insidious six’s. And pulses usually diminished bilaterally. Embolism = sudden, severe pain, unilaterally. How is the presentation of DVT different from that of arterial occlusion? - Correct answer DVT = pain, edema of lower extremity, warmth to touch (vs. coolness), dull/aching pain (vs. sudden/severe pain), present pulses (vs. uselessness). How does iron poisoning present? - Correct answer One of most common causes of death by poisoning. Six’s: bad pain, n/v, diarrhea, *hematemesis*, vasodilation -> hypotensive shock & *anion-gap metabolic acidosis* (poor perfusion, lactic acid accumulation). How to diagnose iron poisoning? - Correct answer Serum iron levels. Abdi X-ray: radiopaque pills. How to TX iron poisoning? - Correct answer Whole-bowel irrigation sometimes. Chelation TX with IV *deferoxamine* in moderate-severe cases. How does acute aspirin poisoning present? - Correct answer Tinnitus, fever, hyperpnoea (rasp alkalosis), & metabolic acidosis. Not radiopaque - wouldn't appear on imaging. How do acute & chronic OD on vet a present? - Correct answer Acute -> n/v, blurry vision. Chronic -> increased ICP (*pseudo tumor cerebra*). How does acute vet D toxicity present? - Correct answer Related to hyper CA -> n/v, confusion, polyuria, polydipsia. How does vet K toxicity present? - Correct answer Very rare; can cause hemolytic anemia, hyperbole in infants. What's the most common malignancy of the thyroid? - Correct answer papillary thyroid cancer (70%), also with the best prognosis. Which thyroid cancers arise from follicular/epithelial cells? - Correct answer Papillary (70%), follicular (20%), and anaplastic (5%) thyroid cancers. Anaplastic carcinoma has the worst prognosis. What cells does medullary thyroid cancer come from? - Correct answer Par follicular C- cells (classically producing *calcitonin*). It's typically a component of MEN 2a & 2b. What are most types of head & neck cancers? - Correct answer Squamous cell carcinomas What is the process behind vitiligo (DE pigmented areas completely lacking melanocytes)? - Correct answer possibly genetic. Autoimmune process directed against melanocytes. Other autoimmune d/o's asset with vitiligo are: *pernicious anemia*, autoimmune thyroid ds (Graves' or chronic autoimmune thyroiditis), DM type 1, primary adrenal insufficiency, hypopituitarism, & alopecia aerate. How does digoxin toxicity present? - Correct answer Use GI effects (anorexia, n/v), bidirectional Attach, accelerated functional rhythms. Can occur with concurrent *verapamil* use (Dec renal clearance of digoxin). What're some common causes of gastritis? - Correct answer Ethos, stress, & NSAIDs. How does mesenteric ischemia present? - Correct answer severe abdominal pain out of proportion to the clinical exam. What are some risk factors for osteomyelitis? - Correct answer Injection drug use, sickle cell anemia, & immunosuppression. Recent distant site infection (UTI). Six’s: *tenderness to gentle percussion* over the spinouts process of involved vertebra. Pain not relieved w/rest. Ds: MRI. TX: long-term IV ax’s with or without surgery. How does enclosing spondylitis present? - Correct answer Pain (insidious onset & intermittent at first) & progressive limitation of back motion. Young men. Six’s improve as day progresses. How does lumbar spinal stenosis present? - Correct answer Narrowing of spinal canal with compression of 1+ spinal roots -> back pain radiating to buttocks/thighs, worse when walking, improved w/lumbar flexion, numbness & paresthesia’s. Ds: MRI. What's dissociation? - Correct answer Neurotic defense mechanism: blocking off disturbing thoughts/feelings from consciousness to avoid emotional upset. What's conversion? - Correct answer Defense mechanism in which emotional conflicts are transformed into physical (often neurological) six’s. Ex: being struck mute after learning distressing news. What're the six’s of galactosemia (infants)? - Correct answer FTT, bilateral cataracts, jaundice, convulsions, hepatomegaly, & hypoglycemia. Caused by *galactose-1- phosphate luridly transferees deficiency*. Inc. risk of *E coli neonatal sepsis*. TX: eliminate GA lactose from diet. How does galactokinase deficiency present? - Correct answer Cataracts. Otherwise asymptomatic. How does luridly triphosphate galactose-4-epimerase deficiency present? - Correct answer rare ds compared with uridyl-transferase deficiency. +hypotonic & nerve deafness. Which Abs’ are most prevalent in Hashimoto's thyroiditis? - Correct answer Anti-TPO (thyroid peroxidase) & anti-thyroglobulin Abs’ (ant microsomal Abs’) What immunoglobulins are present in Graves' disease? - Correct answer Thyroid- stimulating If’s (TSI) -> stimulate TSH receptors on thyroid follicular cells -> thyroid hormone overproduction. What Abs’ are present in primary biliary cirrhosis? - Correct answer Anti-mitochondrial Abs’ What's the difference in presentation btw Guillain-Barre and tick-borne paralysis? - Correct answer Ticks vs. GBS: progressive ascending paralysis *over hrs.-days* (vs. *days-wks.*), usually normal sensation (vs. mildly/grossly ban), & normal CSF (vs. *albumino-cytologic dissociation* i.e., high protein with few cells). TX for tick: remove tick. TX for GBS: IVIG or plasmapheresis. What does the flow-volume loop look like in fixed upper airway obstruction (ex: laryngeal edema)? - Correct answer Flow limitation with both inspiration (below x-axis) and expiration (above x-axis). Pretty symmetric about x-axis. What does the flow-volume loop look like in pneumothorax? - Correct answer Dec ventilation of one hemi thorax -> restrictive pattern on flow-volume testing -> overall volume of flow is decreased, while expiratory flow rate may be increased. What does the flow-volume loop look like in pulmonary edema? - Correct answer Small airway obstruction similar to that seen in asthma -> Dec airflow during passive exhalation -> "scooped out" pattern on exhalation. What does the flow-volume loop look like in a panic attack? - Correct answer Normal. How does lithium intoxication present? - Correct answer Tremors, ataxia, altered mental status, n/v, & hypotension. No muscle rigidity as in NMS. Responds to IVF (SBP > 100mmHg) -> bad CT scan -> surgery depending on grade of injury. What is HELLP syndrome in pregnancy? - Correct answer Pre-ex + Hemolysis, Elevated Liver enzymes, & Low Platelet count. Can have RUQ pain 2/2 distention of liver capsule. Normal PT & PTT. What is idiopathic (immune) thrombocytopenic purport (ITP)? - Correct answer autoimmune destruction of platelets -> bleeding from skin & gums. With HELLP syndrome, when should you NOT deliver? - Correct answer for pregnancies <34wks GA (fetal lung immaturity) with fetal well-being ascertained & mother stabilized. In this case, consider TX with dexamethasone, and delivery when fetus reaches 34 wks. or lungs become mature. How would you diagnose rheumatic fever using the Jones criteria? - Correct answer Ds = preceding group A Strep infection + 2 major criteria or 1 major and 2 minor criteria. Major criteria: migratory polyarthritis, caritas, subcutaneous nodules, erythema marinate, Sydenham chorea. Minor criteria: arthralgia, fever, elevated acute phase reactants (CRP, ESR), prolonged PR interval on EKG. How to diagnose juvenile rheumatoid arthritis? - Correct answer Arthritis present for >6 wks. Can have systemic six’s like rash, but rash wouldn't be migratory (vs. rheumatic fever). How does eosinophilia PNA usually present? - Correct answer Gradual six onset. Asthma-like six’s for several days - diffuse wheezes, fine inspiratory crackles suggestive of bronchial & interstitial involvement. Peripheral eosinophilia. How does leukocytoclastic vacuities usually present? - Correct answer prominent cutaneous manifestations. Six’s of possibly palm infiltrates & pleural effusions are slow to evolve. What kind of neuron disorder is asset with congenital heart disease (esp. R->L shunts)? - Correct answer these kids are at ink risk for developing *brain abscesses* What are some risk factors for developing a brain abscess? - Correct answer congenital heart ds, head trauma, infections of jaw/mouth, and infections of face/scalp, meningitis, & cranial instrumentation. Management of a par pneumonic effusion? - Correct answer Use caused by PNA. May be sterile initially and resolve with ax, but sometimes pt. will persist in having fever, pleurisy chest pain, & dyspnea. CT scan -> aspiration of fluid -> protein content, gram stain, cell count, cytology (r/o cancer), glucose levels, *pH* (< 7.2 almost always indicates *empyema* -> needs chest tube). *Glucose < 60mg/dl* also needs a chest tube. *Total protein may be elevated in almost all exudative effusions (ex: neoplasm, TB, infection). What are some early six’s of Alzheimer's dementia? - Correct answer Visuospatial deficits (getting lost in own neighborhood), anterograde memory problems, cognitive impairment, & language difficulties (word finding). What are some late six’s of Alzheimer's dementia? - Correct answer Personality/Behavior changes (apathy, agitation), neuropsych changes (hallucinations, change in alertness), no cognitive neuron deficits (myoclonus, seizures), urinary incontinence, & apraxia (difficulty using a comb). Ds of Alzheimer's? - Correct answer Clinical dx: MMSE, neuropsych testing, criteria (2+ cognitive deficits, worsening memory or other cognitive fan, onset >60yo, no other cause). What are some diagnostic features of Lowy body dementia? - Correct answer Progressive cognitive decline/dementia, fluctuating cognition, visual hallucinations (recurrent), *spontaneous Parkinsonism features*, REM sleep disorder (vivid dreams), SPECT/PET showing *low DA transporter uptake in basal ganglia*, etc. What's the most prominent clinical feature of normal pressure hydrocephalus (NPH)? - Correct answer Gait impairment, appears early on. Broad-based, shuffling gait. What happens to the liver in Reye syndrome? - Correct answer Aka fatty liver with encephalopathy, seen only in children <15yo, preceded by URI with flu or varicella in 60-80%. History: extensive fatty vacuolization of liver w/o inflammation. Liver is enlarged w/o icterus. How to TX Reye syndrome? - Correct answer Glucose + fresh frozen plasma + manifold (Dec cerebral edema) What happens to the liver in acute alcoholic hepatitis? - Correct answer Balloon degeneration w/polymorphic cellular infiltrates. Accumulation of fat, protein, & water in hepatocytes -> cell swelling -> necrosis What happens to the liver in acute viral hepatitis? - Correct answer Pan lobular mononuclear infiltration w/hepatic cell necrosis (bridging necrosis) What are Chuffer cells? - Correct answer Specialized macrophages of the liver What happens to the liver in chronic active hepatitis? - Correct answer mononuclear inflammatory cells that extend from portal areas to disrupt the limiting perioral plate of hepatocytes undergoing necrosis (piecemeal necrosis) -> perioral septal fibrosis -> bridging fibrosis What happens to the liver in primary biliary cirrhosis? - Correct answer Heavy lymphocytic infiltrate in the portal tracts, asset w/inflammatory granulomatous destruction of small & medium intrahepatic biliary ducts How can dipyridamole or adenosine (coronary vasodilators) be used to dx ischemic heart ds? - Correct answer in normal pts.: they ink coronary blood flow 3-5x. In pts. W/ischemic heart ds: redistribution of coronary blood flow to non-diseased areas vs. diseased (*coronary steal*), b/c diseased vessels distal to obstruction can't dilate any more What's pseudo gout? - Correct answer the acute form of calcium pyrophosphate dehydrate (CPPD) crystal disease. Attacks often occur in setting of trauma, surgery, or medical illness. Synovial fluid will show: rhomboid, positively birefringent crystals. Imaging will show chondrocalcinosis (calcified articular cartilage). What does synovial fluid with hydroxyapatite mean? - Correct answer Degenerative osteoarthritis (hydroxyapatite is completed CA in their cartilage) What do monosodium uprate crystals (in gout) look like? - Correct answer Needle- shaped, negatively birefringent Where is calcium oxalate found? - Correct answer most common crystal found in renal calculi How to manage disk herniation? - Correct answer conservatively (4-6 wks.): early mobilization & NSAIDs & muscle relaxants. If it doesn't resolve, imaging (MRI and CT with or w/o contrast pyelography). If significant or rapidly progressive neuron deficit (foot drop, etc.), emergent surgical decompression. When would you use a plain roentgenogram? - Correct answer those with osteoporosis or possible vertebral crush fractures How to TX cellulitis with systemic signs? - Correct answer IV *nafcillin or cefazolin*. It's usually caused by beta-hemolytic Strep and Staph aurous. Tine pedals (fungal foot infection) may be a portal of entry for lower leg cellulitis. What are some common or serious complications following rhinoplasty? - Correct answer Common: patient dissatisfaction, nasal obstruction, epistaxis. Serious: septal perforation (poor blood supply in septum), use from septal hematoma or septal abscess -> whistling noise during respiration. Complications of acute pancreatitis? - Correct answer Pleural effusion, pancreatic psuedocyst/abscess/necrosis, ileus, ARDS, & renal failure What's the ALT level in biliary pancreatitis? - Correct answer ALT > 150 units/L Besides gallstones & chronic Ethos use, what are other causes of acute pancreatitis? - Correct answer Hyperlipidemia, certain drugs (*didanosine, azathioprine, valproic acid*), infections (*CMV, Legionella, Aspergillums*), trauma, & iatrogenic (post-ERCP) How does mesenteric ischemia usually present? - Correct answer Severe acute per umbilical bad pain that's out of proportion to findings on physical exam. Risk factors: advanced age, diffuse atherosclerosis, alular abnormality, cardiac arrhythmias, or recent MI. How long should you continue an antidepressant if treating for a single episode of major depression? - Correct answer *6 moss* following the patient's response How does cirrhosis predispose to upper GI bleeding? - Correct answer via varies, portal gastropathy, PUD, and Mallory-Weiss tears (retching). How can upper GI bleeding precipitate hepatic encephalopathy in pts. with cirrhosis? - Correct answer Upper GI bleeding -> ink delivery of blood-derived protein to small bowel ->-> hepatic encephalopathy TX: lactulose What are treatments for bleeding varies? - Correct answer Sclerotherapy, endoscopic band ligation, & surgery. Last resort would be a Porto systemic shunt (connecting portal venous system to systemic venous system) via surgery or TIPS procedure. When is surgery for PUD indicated? - Correct answer Perforation, gastric outlet obstruction, or bleeding that can't be controlled with endoscopic TX. How to treat nonbreeding varies? - Correct answer Nonselective beta-blockers like propranolol. What virus causes measles? - Correct answer Paramyxovirus What's the course of measles? - Correct answer Exposure (resp droplets) -> prodrome after 10 days (*coryza/fever, conjunctivitis, cough*) -> Koplik's spots (buccal mucosa) -> skin rash (erythematous maculopapular from face to entire body). What are the complications of measles? - Correct answer PNA, vit A deficiency, & bronchiectasis. Late: immunosuppression & subacute sclerosing panencephalitis. What infection is caused by herpes virus 6, and what's it look like? - Correct answer Roseola infantum. Sx's: fever -> maculopapular rash from trunk to extremities after 4-5 days. What infection is caused by togavirus, and what's it look like? - Correct answer Rubella (aka "3-day measles"). Sx's: erythematous maculopapular rash (fine, pink, spreads from face to trunk in 3 days) w/generalized LAD. Also slight fever, polyarthritis, & polyarthralgia. What does chicken pox look like? - Correct answer Fever, malaise, & anorexia -> rash from trunk to extremities -> vesicles -> scabs. What infection is caused by parvovirus, and what's it look like? - Correct answer Erythema infectiosum. Ex's: erythema of cheeks -> erythematous rash from arms to trunk & legs. What are some behavioral therapies for specific phobias? - Correct answer Flooding, biofeedback, relaxation therapy, & exposure desensitization. What's 1st line tx for generalized anxiety d/o? - Correct answer Buspirone When should you screen/treat asymptomatic bacteriuria in pregnant women? - Correct answer Between 12-16 wks GA -> significantly decreases incidence of symptomatic UTIs, LBW babies, and preterm deliveries. When should you screen for HIV in pregnant women? - Correct answer At first prenatal visit, and again in the 3rd trimester before 36 wks for those at high risk of acquiring HIV. What are major risk factors for squamous cell carcinoma of the esophagus? - Correct answer Smoking, EtOH, dietary deficiency of beta-carotene, vit B1, zinc, selenium, environmental viral infections, toxin producing fungi, hot food/beverages, pickled veggies, & food rich in N-nitroso compounds. What are major risk factors for adenocarcinoma of the esophagus? - Correct answer Barrett's esophagus, obesity, high dietary calorie/fat intake, smoking, & medications promoting GERD. What's placenta previa? - Correct answer Insertion of the placenta in a way that obstructs the internal cervical os partially/completely -> may cause bleeding as the inferior uterus develops & stretches the placenta. Cause of 20% of all cases of antepartum bleeding. Typically painless. What are risk factors for placenta previa? - Correct answer Multiparity, advanced maternal age, prior C/S, smoking, multiple gestation, & prior placenta previa. How can placental abruption present? - Correct answer Variable clinical presentation: pts may be asymptomatic with intrauterine fetal death, or may present with dark red vaginal bleeding & painful uterine contractions. What's vasa previa? - Correct answer Rare condition in which fetal blood vessels cross the fetal membranes in the lower uterus btw fetus & internal cervical os. Painless antepartum hemorrhage, but rapid deterioration of fetal heart tracing (hemorrhage is from fetus). How does PE (w/subsequent minor/massive pulmonary infarction) usually present? - Correct answer *Pleuritic chest pain* in the setting of prolonged immobility, *hemoptysis*, dyspnea, *tachycardia*, & *OCP use*. What does dermatomyositis look like? - Correct answer Autoimmune condition causing a proximal extensor muscle inflammatory myopathy & cutaneous findings (violaceous poikiloderma over various regions of body). Face + periorbital edema = heliotrope sign. Chest & lateral neck = shawl sign. Knuckles, elbows, & knees = Gottron's sign. Lichenoid papules overlying joints = *Gottron's papules* (pathognomonic). What's the classic Ab in dermatomyositis? - Correct answer Anti-Mi-2 Ab (against *helicase*) What condition is associated with dermatomyositis? - Correct answer Malignancy (>10%), usually: *ovarian* > breast/lung/female GU in women, and colorectal/lung in men. What inflammatory dz's are associated with aortic aneurysm? - Correct answer Behcet syndrome, Takayasu arteritis, giant cell arteritis, ankylosing spondylitis, rheumatoid arthritis, relapsing polychondritis, & reactive arthritis. What autoimmune dz's are associated with renal failure? - Correct answer SLE, scleroderma, Goodpasture syndrome, relapsing polychondritis, & Wegener's granulomatosis w/polyangiitis. What inflammatory dz's are associated w/alveolar hemorrhage? - Correct answer Goodpasture syndrome, Wegener's granulomatosis w/polyangiitis, polyarteritis, Churg- Strauss syndrome, Behcet syndrome, & antiphospholipid antibody syndrome. What inflammatory/depositional dz's are associated w/carpal tunnel syndrome? - Correct answer Rheumatoid arthritis, sarcoidosis, & amyloidosis. Which organisms are most commonly responsible for brain abscesses? - Correct answer Aerobic/anaerobic Strep (60-70%) and Bacteroides species (anaerobes) (20- 40%). How to work up vertigo? - Correct answer Differentiate vertigo from presyncope or disequilibrium -> classify as central (CN 8) or peripheral (*Meniere's* is abnl accumulation of endolymph in inner ear). How do disorders of the middle ear (vs inner ear) usually present? - Correct answer Middle ear d/o -> hearing loss. Inner ear d/o -> vertigo. What's Wallenberg syndrome? - Correct answer Lateral medullary infarction. Sx's: intense vertigo, gaze abnl, limb ataxia, sensory loss, & Horner's syndrome. What are some precipitating factors of osteonecrosis (avascular necrosis)? - Correct answer Chronic corticosteroid use, chronic EtOH use, trauma, antiphospholipid syndrome. Caused by disruption of vasculature to affected bone -> bone & bone marrow infarction. MRI is the most sensitive test (Xray may not show anything). What causes trochanteric bursitis (inflammation of the trochanteric bursa)? - Correct answer Caused by friction btw the tendons of the gluteus medius & tensor fascia lata over the greater trochanter of the femur. -> Pain over the lateral hip, worsened by palpation. May interfere w/sleeping. What's Leriche syndrome? - Correct answer *Aortoiliac* occlusion. Sx's: low back, hip, buttock, & thigh claudication + impotence + atrophy of lower extremities. Weak femoral pulses. ~Bruit over iliac/femoral arteries. What's Shy-Drager syndrome (multiple system atrophy)? - Correct answer Degenerative disease characterized by: 1) Parkinsonism, 2) autonomic dysfxn (orthostatic hypotension, incontinence, impotence, etc), & 3) widespread neuro signs. *Bulbar dysfxn & laryngeal stridor may be fatal. Tx: intravascular volume expansion w/*fludrocortisone*, salt supplementation, a- adrenergic agonists, & leg constrictors. What causes idiopathic orthostatic hypotension? - Correct answer 2/2 degeneration of postganglionic sympathetic neurons - pathology confined to autonomic nervous system. What's the triad of Horner's syndrome? - Correct answer Anhidrosis (no sweating), miosis (pupil constriction), & ptosis. What's Riley-Day syndrome (familial dysautonomia)? - Correct answer AR dz seen in Ashkenazi Jews. Gross dysfxn of the autonomic nervous system w/severe orthostatic hypotension. What's polycythemia vera? - Correct answer It's a myeloproliferative disorder (of RBCs, basophils) -> splenomegaly, pruritis with hot baths (histamine release from inc basophils), inc catabolism/turnover of purines -> uric acid overproduction -> gout in 40% of pts. What are some organ-specific consequences of hemochromatosis (iron overload in organs)? - Correct answer Hepatic dysfxn -> cirrhosis, diabetes, hypogonadism, skin pigmentation, & arthritis. How does Lesch-Nyhan syndrome present? - Correct answer Genetic deficiency of enzyme *HPRT* -> overproduction of uric acid, behavioral problems (self-injury), and neuro disability, in childhood. Why is portal hypertension associated with severe pruritis? - Correct answer Inc serum concentrations of bilirubin. What is injured in penile fracture? - Correct answer The *tunica albuginea*, which invests the corpus cavernosum, is torn. Tx: emergent urethrogram to assess for urethral injury + emergent surgery to evacuate the hematoma & mend the tunica albuginea. What does circumcision treat? - Correct answer Phimosis, paraphimosis, & Zoon's balanitis. How does intussusception usually present? - Correct answer Most common cause of intestinal obstruction in children 6-36 mos. Lead point sometimes from viral GI infection, Meckel's diverticulum, polyps, or hematomas (HSP) - though most times no identifiable lead point. Usu involves ileocolic jxn. Intermittent *telescoping* of 1 bowel segment into another -> periodic pain, emesis, tubular "sausage-shaped" mass, "currant jelly" stools. Dx: U/S. Tx: immediate air/contrast enema reduction - emergency! What does a technetium-99m pertechnetate scan ("Meckel's scan") detect? - Correct answer Detects gastric mucosa, which is present in ~1/2 of Meckel's diverticula. How does meconium ileus usually present? - Correct answer Bilious vomiting, failure to pass meconium at birth, hx of polyhydramnios, family hx cystic fibrosis, & typical appearance on abd Xrays (dilated small bowel, ground glass appearance in lower abd). Complications include *intestinal perforation*. *This condition is diagnostic of cystic fibrosis. What is meconium plug syndrome? - Correct answer Similar presentation to meconium ileus (bilious vomiting, failure to pass meconium at birth, hx of polyhydramnios), but occurs in *non-cystic fibrosis* babies. No complication of intestinal perforation. Why is atropine contraindicated in acute glaucoma? - Correct answer Atropine can dilate the pupil and worsen the glaucoma (inc intraocular pressure). How to manage acute glaucoma? - Correct answer It's a medical emergency. Tx: narcotics for pain, mannitol/timolol/acetazolamide/pilocarpine to dec intraocular pressure. 1st line = *IV mannitol* (osmotic diuretic that works immediately). Pilocarpine (topical) opens the canals of Schlemm -> drainage of aqueous humor. *Avoid atropine (dilates pupil, worsens glaucoma)! How does exertional heat stroke present? - Correct answer In extreme heat & humidity: temp > 105F, *altered mental status*, hypotension, tachycardia, & tachypnea. Pts often aren't volume depleted. Large blood on UA dipstick w/o RBCs by microscopy suggests rhabdomyolysis. Pathophys: thermoregulatory center fails to dissipate heat at the rate necessary to maintain euthermia. What's the underlying pathophys of malignant hyperthermia? - Correct answer Uncontrolled efflux of Ca from sarcoplasmic reticulum. Rare AD disorder. Occurs in these pts after administration of anesthetic drugs *halothane & succinylcholine*. What's the pathophys of heat exhaustion? - Correct answer Inadequate sodium & water replacement during physical activity -> inability to maintain cardiac output -> heat exhaustion. Core temp usu < 104F. No significant CNS dysfxn. What's the usual presentation of hypertrophic cardiomyopathy? - Correct answer Young, physically active individuals -> cardiac outlet obstruction -> dyspnea, palpitations, syncope, sudden cardiac death. What is pulsus paradoxus? - Correct answer Fall in systolic arterial pressure >10 mmHg during inspiration. Most commonly seen in cardiac tamponade, but also seen in COPD & asthma b/c of great drop in intrathoracic pressure (inc negative P). How does pulsus paradoxus occur? - Correct answer Relatively low LV preload or end diastolic volume during inspiration -> dec stroke volume -> much reduced systolic bp (>10mmHg) during inspiration. What is the most common congenital obstructive urethral lesion in males? - Correct answer Posterior urethral valves = abnl folds in posterior urethral wall that obstruct urine flow. Presents w/distended bladder (midline abd mass) - shows that obstructive lesion is distal to bladder neck. Dx: voiding cystourethrogram (VCUG). What is the urachus? - Correct answer A tubular extension of the allantois that extends from bladder to umbilicus. Normally obliterated during development. Patent urachus -> urachal fistula, cyst, or sinus. What's an omphalomesenteric duct cyst? - Correct answer A focal failure of vitelline duct obliteration. May cause a small midline mass deep to umbilicus. If attached to umbilicus & wall of ileum, the cyst could cause a small intestinal volvulus. What's a missed abortion? - Correct answer A form of spontaneous abortion where the fetus expires in utero but products of conception aren't spontaneously discharged from the uterus. Pts present with loss of pregnancy sx's and no continued increased in uterine size. What are the different types of metatarsus adductus (congenital foot deformity)? - Correct answer Type I: feet that overcorrect both passively & actively into abduction. Tx not necessary, but still assoc w/hip dysplasia. Type II: feet that correct to neutral position w/passive & active mvmts. Tx: orthosis +/- plaster casts. Type III: rigid feet. Tx: serial casts. What are the risk factors for developing melanoma? - Correct answer Genetic: Caucasian, fair complexion. Environ: *XS sun exposure*, tanning beds, PUVA tx, multiple congenital nevi, immunosuppression, family hx, self hx. Does SPF 15-30 sunscreen protect against melanoma? - Correct answer Research says no. But there's evidence of protection against non-melanoma skin cancers (ie, squamous cell carcinoma). What is Turcot's syndrome? - Correct answer AR association btw brain tumors (mostly *medulloblastomas & gliomas*) and FAP or HNPCC (*colon polyps*). Pts w/HNPCC are prone to high-grade gliomas. What is Gardner's syndrome? - Correct answer AD association btw *colon polyps* and prominent extraintestinal lesions (desmoid tumors, sebaceous/epidermoid cysts, lipomas, osteomas, supernumery teeth, gastric polyps, juvenile nasopharyngeal angiofibromas). What is Peutz-Jeghers syndrome? - Correct answer AD association btw *intestinal hamartomatous polyps* and *mucocutaneous melanocytic macules*. What's Cowden syndrome? - Correct answer Aka multiple hamartoma syndrome. Association btw *GI hamartomas* and *breast cancer, thyroid cancer, & nodular gingival hyperplasia*. What do you do with a pt with BPH and elevated creatinine? - Correct answer Suspect urinary obstruction -> *abd U/S* to look for *hydronephrosis*. If present, Foley catheter will help. What's TURP? - Correct answer Transurethral resection of the prostate - surgical intervention for BPH with urinary obstruction. How does craniopharyngioma usually present? - Correct answer Benign tumors from Rathke's pouch. Bimodal age distribution (children & 55-65yo). Located above the *sella tursica*, multiple cysts, grows gradually over yrs, usually presents with *hypopituitarism* (retarded growth in kids, sexual dysfxn/amenorrhea in adults). Compresses the optic chiasm -> *bitemporal blindness*. Dx: MRI/CT. Tx: surgery and/or radiotherapy. How do alcoholic & diabetic neuropathies usually present? - Correct answer Usually distal and symmetric (glove & stocking distribution). How does optic neuritis present? - Correct answer May be monocular or binocular, depending on cause. Usu assoc w/loss of central vision (*scotomas*) and afferent pupillary defect. What murmur can ascending aortic aneurysm be associated with? - Correct answer Aortic regurgitation (diastolic murmur) What is chlordiazepoxide? - Correct answer Librium, a BDZ. Common choice of tx for alcohol withdrawal. What are some side effects of vancomycin? - Correct answer Nephrotoxicity, ototoxicity, and "red man syndrome." What are signs of opioid withdrawal? - Correct answer Pupil dilation, piloerection, dysphoria, myalgias, abd pain. How does juvenile angiofibroma (JNA) usually present? - Correct answer Any adolescent who presents with nasal obstruction, visible nasal mass, and frequent nosebleeds is considered to have JNA unless proven otherwise. Benign growth, but can erode and locally invade. Potentially dangerous b/c made up of many blood vessels which can readily bleed. Surgical removal is difficult b/c tumor is unencapsulated and may be deeply invasive. Recurrence is common. What's the usual course of acute hep B infection? - Correct answer >90% of adults will recover completely. A minority of adults will develop chronic hep B infection. 0.1-0.5% will develop fulminant hepatic failure (FHF) - hepatic encephalopathy w/in 8 wks of onset of acute liver failure. These pts are high-priority for liver transplant, the only cure. What are contraindications for liver transplantation? - Correct answer Irreversible cardiopulmonary dz causing prohibitive risk, incurable or recent (<5yrs) cancer outside the liver, and active EtOH/drug abuse. What is chronic granulomatous disease (CGD)? - Correct answer Defect of phagocytic cells due to dysfxn of NADPH oxidase enzyme complex -> recurrent/uncontrolled infections w/catalase-positive organisms (*Staph aureus, Serratia, Burkholderia, Klebsiella, Aspergillus*). Dx: nitroblue tetrazolium slide test, flow cytometry, or cytochrome C reduction. Tx: daily TMP-SMX, gamma-interferon 3x/wk. What's Chediak-Higashi syndrome? - Correct answer Decreased degranulation, chemotaxis, & granulopoiesis. Multisystem d/o. Dx: neutropenia & giant lysosomes in neutrophils. Tx: daily TMP-SMX & ascorbic acid. What are leukocyte adhesion syndromes? - Correct answer Failure of innate host defenses against microorganisms 2/2 defective tethering, adhesion, & targeting of myeloid leukocytes to sites of microbial invasion. Hallmark: neutrophilia w/o polymorphs in infected tissue/pus. Hx of delayed separation of umbilical cord, recurrent bacterial infections, necrotic skin lesions, etc. What's hyper-IgE (Job's) syndrome? - Correct answer High serum IgE levels, eosinophilia -> chronic pruritic dermatitis, recurrent Staph infections (skin & resp tract), coarse facial features, bone fractures. What happens in Cushing's syndrome? - Correct answer High *cortisol* levels -> vasoconstriction (HTN), insulin resistance (hyperglycemia), and mineralocorticoid activity (hypokalemia). Also: central adiposity, skin thinning, weight gain, psych problems (sleep, depression, psychosis). What causes Cushing's syndrome? - Correct answer Adrenal cortical hyperplasia, ACTH-producing pituitary adenoma (Cushing's disease), ectopic ACTH production, or exogenous steroids. What happens after a stroke involving the ventral postero-lateral (VPL) nucleus of the thalamus? - Correct answer VPL transmits sensory info from the contralateral body. Classic presentation of stroke: contralateral hemianesthesia, transient hemiparesis, athetosis (ballistic mvmts), thalamic pain phenomenon (dysesthesia/pain of area affected by sensory loss). What's the definition of hazard ratio? - Correct answer Ratio of an event rate occurring in the tx group compared to an event rate occurring in the non-tx group. What is cluster analysis for an expt? - Correct answer Grouping of diff data points (ex: individuals) into similar categories - usu involves randomization at the level of groups rather than individuals. What's a parallel study design? - Correct answer Randomizes one tx to one group and a diff tx to the other group. There are usually no other independent variables measured. What test is usually used to screen for macular degeneration? - Correct answer The grid of parallel vertical & horizontal lines -> one of the earliest findings in macular degeneration is distortion of straight lines such that they appear wavy. Fine visual acuity will be affected. Ophtho exam might also find drusen deposits in the macula. *False positive with bacterial overgrowth (bacteria ferment the D-xylose -> reduces absorption & urinary excretion). What is D-xylose? - Correct answer It's a simple sugar (monosaccharide) that doesn't need any digestive process before being absorbed by *proximal* small intestines into venous circulation -> urinary excretion. What is modafinil approved to treat? - Correct answer Narcolepsy How do you manage cocaine-induced coronary vasoconstriction & MI? - Correct answer 1) IV BDZs for BP & anxiety, 2) Aspirin, 3) Nitroglycerin & Ca channel blockers for pain, 4) Immediate cardiac cath w/reperfusion when indicated (ST elevation or persistent chest pain despite meds). *Beta-blockers are contraindicated (can cause unopposed alpha adrenergic stimulation)! Fibrinolytics not preferred due to inc risk of intracranial hemorrhage. What's obesity hypoventilation syndrome (aka Pickwickian Syndrome)? - Correct answer Obesity impedes the expansion of the chest/abd wall during breathing -> inc WOB & dec resp drive -> underventilation of lungs during all hours ->-> pulm HTN and RV failure can result. HOw does meningococcal meningitis usually present? - Correct answer Sx's of bacterial meningitis: sudden onset of fever, stiff neck, HA, nausea, myalgias. Sx's of meningococcus: hypotension, tachycardia, *myalgias*, *purpuric skin lesions*. How does RMSF usually present? - Correct answer CSF findings are more consistent with a viral meningitis (vs bacterial) picture. Rash begins on ankles/wrists and spreads centrally & to palms/soles. How does encephalitis present differently from meningitis? - Correct answer Encephalitis: altered mental status, fever, focal neuro deficits, NO nuchal rigidity or other signs of meningitis. What are the arboviruses and what kind of illness do they cause? - Correct answer Arboviruses include eastern equine, western equine, St. Louis, & West Nile virus. They typically cause encephalitis. What is a relatively common (10%) complication of epidural anesthesia? - Correct answer Sympathetic fiber block -> vasodilation of lower extremity vessels -> venous pooling & hypotension How to manage a threatened abortion? - Correct answer Dx: ascertain fetus is present and alive with U/S. Mgmt: reassurance and follow-up U/S after 1 wk. How to tx ectopic pregnancy? - Correct answer Methotrexate How to tx a missed abortion? - Correct answer Dilation & (suction) curettage, b/c the cervix is closed and expulsion of the expired fetus doesn't always occur spontaneously. What do you do when you suspect traumatic hemorrhage of a solid intraabdominal organ? - Correct answer Stable pts: CT abdomen. Unstable pts: FAST exam (focused assessment w/sonography for trauma) -> if inconclusive, diagnostic peritoneal lavage (DPL) to determine the need for emergent laparotomy. What would cause post cholecystectomy pain? - Correct answer Generally functional etiology (dx of exclusion), sphincter of Oddi dysfxn (tx w/ERCP sphincterotomy), or common bile duct stones (r/o through ERCP & U/S). How does a meniscal injury usually present? - Correct answer Twisting injury with the foot fixed, medial > lateral, popping sound followed by severe pain at time of injury, effusion not until many hrs later (meniscus not directly perfused). McMurray's sign for medial meniscus tear = palpable/audible snap while slowly extending leg at the knee from full flexion while applying tibial torsion. How does ACL tear usually present? - Correct answer Forceful hyperextension to knee or a noncontact torsional injury of knee during deceleration. Effusion rapidly following injury. Lachman's test, anterior drawer test, & pivot shift test for clinical dx. How does PCL tear usually present? - Correct answer "Dashboard injury" - forceful posterior-directed force on tibia with knee flexed at 90 deg. Posterior drawer test, reverse pivot shift, & posterior sag tests for clinical dx. How does racemic epinephrine help with croup pts? - Correct answer Alpha-adrenergic effects: reduce bronchial secretions & mucosal edema. Beta-adrenergic effects: promote smooth muscle relaxation. *Always give a trial of epi in croup before intubation. How does anemia of prematurity usually present in babies? - Correct answer Premature at birth, 2-3 mos old, pallor, tachypnea, tachycardia. Tx: iron supplementation. When are solid foods usually introduced in a baby's diet? - Correct answer Veggies, egg, meat are usually introduced at 4 mos old. When are fruit juices usually introduced in a baby's diet? - Correct answer ~5-7 mos old. Good sources of vitamins. How does a complex partial seizure usually present? - Correct answer Brief (few minutes) episodes of impaired consciousness, failure to respond to various stimuli during the episode, staring spells, automatisms (lip smacking, swallowing, etc), and post-ictal confusion. EEG usually normal or may show brief discharges. *Hyperventilation* during EEG can't simulate a complex partial seizure (unlike in absence seizures). How does a typical absence seizure usually present? - Correct answer Brief (seconds) periods of impaired consciousness, ~automatisms, no post-ictal state. Hyperventilation during EEG reveals generalized 3Hz spike-and-wave pattern on normal background. How does an atypical absence seizure usually present? - Correct answer Lasts longer than typical absence seizure. EEG pattern is slow spike-and-wave activity with frequency <2.5Hz. How does juvenile myoclonic epilepsy usually present? - Correct answer Adolescents who have unilateral or bilateral myoclonic jerks. Sx's in the morning, may be precipitated by sleep deprivation. How does Lennox-Gastaut syndrome usually present? - Correct answer Childhood, seizures of multiple types, impaired cognitive fxn, slow spike-and-wave activity on EEG. What to do with diarrhea in HIV+ pts? - Correct answer Stool culture, examination for ova & parasites, and test for C. diff toxin. For those with persistent diarrhea and negative stool exam, consider colonoscopy and biopsy of mucosa and/or ulcers. How does MS usually present? - Correct answer Relapsing-remitting various neuro deficits. Typically, optic neuritis (painful loss of vision), diplopia, sensory deficits, motor weakness, bladder/bowel dysfxn, neuropsych disturbances. Women of childbearing age. Dx: MRI with and w/o gadolinium. LP for oligoclonal bands. How should you manage the airway of a burn victim? - Correct answer Treat initially with high-flow O2 via non-rebreather mask, and early intubation if there's evidence of thermal damage to the upper airway - esp to prevent upper airway obstruction later on by edema. What is N-acetylcysteine used for? - Correct answer Antidote for acetaminophen- induced hepatotoxicity, as a mucolytic agent in cystic fibrosis, to prevent radiocontrast nephropathy in pts with renal insufficiency undergoing CT scans, and may also treat CO poisoning (via case reports). How does warfarin work? - Correct answer Inhibits synthesis of vit K-dependent clotting factors II, VII, 9, & X (prothrombin), as well as vit K-dependent anticoag proteins C & S. It can therefore cause a false-positive test for protein S deficiency. How does heparin work? - Correct answer Activates antithrombin III -> inactivates thrombin, factor IXa, & factor Xa. How to tx suspected PCP PNA in an HIV+ pt with CD4 count <200? - Correct answer TMP-SMX, along with prednisolone when PaO2 <70 mmHg. What are some metabolic consequences of pts with OSA and obesity hypoventilation syndrome (leading to chronic resp failure)? - Correct answer Resp acidosis -> inc bicarb retention -> dec chloride reabsorption. Also, volume overload -> hypervolemic hypoNa. When would you perform a VCUG to identify structural abnormalities predisposing to UTIs? - Correct answer If pt is <10yo, male, or a prepubertal non-sexually active female. What happens with hypercalcemia? - Correct answer HyperCa -> induces urinary salt wasting -> volume depletion -> exacerbates hyperCa by impairing renal clearance of Ca -> vicious cycle. Tx: normal saline at 200ml/hr with goal urine output of 100-150 ml/hr. Plus *calcitonin* + *bisphophonate (zoledronic acid)* to rapidly reduce serum Ca. Continue bisphosphonate long-term for hyperCa of malignancy. How do you treat vit D-related hypercalcemia (ie, sarcoidosis & lymphoma)? - Correct answer Glucocorticoids Which tumors metastasize most commonly to the liver? - Correct answer Primary tumors of the GI tract, lung, and breast. How does hyperPTH usually present? - Correct answer Elevated Ca, low P -> constipation, fatigue, XS urination, abd pain, urinary stones (Ca phosphate), mental status changes, osteoporosis, pseudogout of knee (calcium pyrophosphate dihydrate crystals). Synovial fluid will reveal rhomboid-shaped, positively birefringent crystals. How does adult polycystic kidney disease usually present? - Correct answer AD progressive kidney disease. Recurrent flank pain, renal infections, & nephrolithiasis, inc risk of RCC -> end stage renal dz in most pts by 60yo. How does acute glomerulonephritis usually present? - Correct answer Proteinuria, edema (usu periorbital), HTN, renal insufficiency. How does Schistosoma haematobium infection usually present? - Correct answer Dysuria & urinary frequency -> gross hematuria & bladder pain later on. Common cause of hematuria in Middle East & Africa. Snail = intermediate host. What's the most common cause of painless hematuria in adults? - Correct answer Bladder tumors. How does Coxsackie viral infection usually present? - Correct answer Fever, HA, stiff neck (ie, aseptic meningitis). Can also cause myocarditis, hand-foot-and-mouth dz, and herpangina (vesicles on the hard palate). Which regions are hyperendemic for Giardia? - Correct answer Nepal and certain mountainous areas of northern & western US. How does antiphospholipid antibody syndrome usu present in pregnant women? - Correct answer Commonly causes a false positive VDRL (confirm w/FTA-ABS test), hx of arterial/venous thromboses, hx of spontaneous abortions, thrombocytopenia & prolonged PTT. Tx: LMW heparin during pregnancy (warfarin contraindicated in pregnancy). What are the K+ sparing diuretics? - Correct answer Spironolactone, eplerenone, triamterene, & amiloride - act on principal cells in cortical collecting tubule. What's the dx for generalized anxiety d/o? - Correct answer XS anxiety about multiple things, plus 3+ sx's for >6 mos: impaired sleep, poor concentration, easy fatigability, irritability, muscle tension, & restlessness. Which neutrophilic fxn is impaired in chronic granulomatous disease (CGD)? - Correct answer Intracellular killing. Other neutrophilic fxns (chemotaxis, phagocytosis, & degranulation) are intact. So a Gram stain of infection will show neutrophils filled with bacteria. What's adenosine deaminase deficiency? - Correct answer AR cause of severe combined immunodeficiency (SCID). PCP, Candida, Parainfluenza, & herpesvirus infections are most common. Lymphopenia is a classic finding. What happens in complement overconsumption? - Correct answer Occurs when large quantities of Ab-Ag complexes are present, as in vasculitis, glomerulonephritis, & certain CT dz's. Classic example is SLE. What does defective opsonization lead to? - Correct answer C3 is the major opsonin of the complement system. C3 deficiency -> recurrent infections with encapsulated bacteria (Pseudomonas, etc), starting shortly after birth. What does B-cell maturational arrest lead to? - Correct answer It's the best-described cause of agammaglobulinemia, the most common primary immunodeficiency. Leads to poor Ab production -> high risk for infections w/encapsulated bacteria (require Ab binding for effective opsonization) -> sinusitis, bronchitis, otitis media, & PNA are common. What virus causes Kaposi sarcoma, and how does it present? - Correct answer HHV-8. Presentation: cutaneous lesions (asymptomatic, elliptical, light brown -> violet colored), homosexual HIV pts, also in colon. How to manage squamous cell carcinoma of the vagina? - Correct answer Most common vaginal cancer. Sx's: vaginal bleeding, malodorous discharge. Dx: biopsy. Tx: without extension to pelvic wall or mets, size <2cm -> surgery. Size >2cm -> radiation tx. *If really old and poor surgical candidate (multiple comorbidities), consider radiation even if size <2cm. What is hyposthenuria? - Correct answer Impairment in kidney's ability to concentrate urine -> nocturia, frequent urination. Found in pts with sickle cell dz, but also common (less severe) in those with sickle cell trait. Thought to result from RBC sickling in vasa rectae of inner medulla -> impairs countercurrent exchange & free water absorption. What kinds of renal d/o's might those with sickle cell develop? - Correct answer Membranoproliferative glomerulonephritis or end-stage renal dz 2/2 focal segmental glomerulosclerosis. Typically manifest w/abnl urinary sediment (RBC casts). How can you tell when food poisoning comes from a preformed exotoxin? - Correct answer Rapid onset of sx's (<6hrs) and often involving vomiting (vs diarrhea). Which organisms are more likely to give food poisoning from preformed exotoxins? - Correct answer Staph aureus: poultry/eggs, meat, salads made w/mayo, cream-filled pastries, milk/dairy. Bacillus cereus: starchy foods (rice, classically reheated fried rice). Why should sunscreen generally be avoided in infants <6 mos? - Correct answer Thin skin and high SA:body ratio -> inc exposure to sunscreen chemicals. What should you be concerned about when a pt has painless jaundice with conjugated hyperbili and markedly elevated alk phos? - Correct answer An intraabdominal malignancy obstructing the biliary system, such as pancreatic adenocarcinoma. Dx: CT abdomen. What is the diagnostic imaging of choice for assessing soft tissue injuries of the knee? - Correct answer MRI - can detect complete vs partial tears, the exact site of ligamentous injury, and associated injuries to other ligaments or meniscus. *Arthroscopy is an invasive procedure, used where MRI is inconclusive or surgery is necessary. How do you manage an MCL tear in the knee? - Correct answer Bracing & early ambulation; surgery is rarely necessary When is cholecystectomy indicated for acute pancreatitis? - Correct answer For all pts with symptomatic gallstones (aka pancreatitis if otherwise asymptomatic) who are medically stable enough for surgery. What's erysipelas? - Correct answer A specific type of cellulitis characterized by inflammation of superficial dermis -> prominent swelling. Classic finding: sharply When does stranger anxiety usually stop? - Correct answer 3yo. After this age, a persistent fear of strangers is usually due to other causes. What's selective mutism? - Correct answer A condition in which children have a fear of situations that call for them to speak (school, formal social gathering, etc). What does theophylline toxicity look like? - Correct answer CNS stimulation (HA, insomnia, SZ), GI disturbance (n/v), and cardiac toxicity (arrhythmia). Theophylline is metabolized by cytochrome oxidase system in liver - these enzymes can be inhibited by concurrent illness or drugs (cimetidine, cipro, erythromycin, clarithromycin, verapamil). What are the two most common peripheral artery aneurysms? - Correct answer Popliteal aneurysm > femoral artery aneurysm (either may be assoc w/AAA) What usually causes femoral vein aneurysm? - Correct answer Femoral vein dilation usually results from AV fistula (typically traumatic) -> venous HTN. How does herpes simplex keratitis usually present? - Correct answer Frequent cause of corneal blindness in the US, usu in adults. Pain, photophobia, blurred vision, tearing, & redness of eye. Hx of prior episodes. Precipitated by XS sun, outdoor occupation, fever, or immunodeficiency. Corneal vesicles & *dendritic ulcers*. Tx: antiviral (oral or topical). How does bacterial keratitis usually present? - Correct answer Contact lens wearers, following corneal trauma or entry of foreign body. Cornea looks hazy with a central ulcer & adjacent stromal abscess. How does herpes zoster ophthalmicus usually present? - Correct answer Elderly pt. Fever, malaise, burning, itching in periorbital region. Vesicular rash in distribution of cutaneous branch of CNV1. How does fungal keratitis usually present? - Correct answer Occurs after corneal injury in agricultural workers or immunocompromised pts. Cornea shows multiple stromal abscesses. How do you a manage a mildly tender breast lump of a young woman with no obvious signs of malignancy? - Correct answer Ask her to return after her menstrual period for re-exam. If mass decreases in size after period, most likely benign. Otherwise, do U/S, FNA biopsy, and/or excisional biopsy. *Mammography usu not helpful here b/c density of breast tissue is high in young women. What type of imaging can confirm the presence, size, and location of an intracranial hemorrhage? - Correct answer Head CT w/o contrast How would you evaluate a couple for infertility, after doing physical exam and getting history? - Correct answer 1) Semen analysis (male factor is 20-30% of infertility causes). 2) Evaluate for anovulation via basal body temperature measurement, serum progesterone level (during mid-luteal phase), & endometrial sampling. 3) More complicated tests: hysterosalpingography, laparoscopy. And serum prolactin level. What's postop endophthalmitis? - Correct answer Most common form of endophthalmitis. Usu occurs w/in 6 wks of surgery. Infection within the eye, particularly the vitreous. Sx's: pain & decreased vision, swollen eyelid/conjunctiva, corneal edema, infection. Tx: based on severity, intravitreal abx injection or vitrectomy. How does conjunctivitis usually present? - Correct answer XS tearing, burning, mild pain, conjunctival & eyelid edema. Vision isn't affected. How does uveitis usually present? - Correct answer Blurred vision with moderate pain, conjunctival injection, & constricted pupils. Keratic precipitates ("mutton fat") and iris nodules may be seen. Assoc w/HLA B27-related conditions. How does cavernous sinus thrombosis usually present? - Correct answer Proptosis, ophthalmoplegia, chemosis, & visual loss. Occurs due to hematogenous spread from an infected, inflamed sinus. How can effect modifiers be distinguished from confounders? - Correct answer Perform a stratified analysis centering on the variable of interest. If the variable is a confounder, there won't be a significant difference in risk btw stratified groups (confounding effects have been removed through stratification). If variable is an effect modifier, there will be a significant difference btw the 2 groups. What's the age group that typically develops imaginary friends? - Correct answer 2-6yo, typically in response to times of change or stress What should you be concerned for with a middle-aged woman with dry eyes, dry mouth, trouble swallowing, and dental caries? - Correct answer Sjogren syndrome: an autoimmune condition with lymphocytic infiltration of the salivary glands -> enlargement & firmness of these glands. Dx: dry mouth & eyes with histo evidence of lymphocytic infiltration of salivary glands or serum Ab's against SSA (Ro) and/or SSB (La). What're the three A's of negative sx's in psychosis? - Correct answer Affective flattening, Asociality, & Alogia (won't talk). What are the majority of lip cancers? - Correct answer Well-differentiated squamous cell carcinomas, which on histo looks like invasive cords of squamous cells with keratin pearls. Also hx of occupational sun exposure. What are clues that a woman is ovulating regularly? - Correct answer Regular menstrual cycles accompanied by midcycle pelvic pain and discharge consistent with ovulation ("egg-like" thickening), & rise in body temperature during the luteal phase. What're some causes of anatomic defects in the uterus or fallopian tubes leading to infertility? - Correct answer Hx of PID, endometriosis, in utero DES exposure, congenital malformations, & other acquired abnormalities. Why are pts with PCOS typically anovulatory? - Correct answer XS serum androgen levels -> suppress GnRH as well as FSH release via feedback inhibition -> suppress ovulation. You can also tell b/c XS androgen can give hirsutism, acne, & virilization. What does serum inhibin B test? - Correct answer Determines ovulatory reserve. What would you suspect in a pt with cupping of the optic disc and loss of peripheral vision? - Correct answer High intraocular pressure - may indicated open angle glaucoma, usu asymptomatic in earlier stages, but more common in AAs, family hx of glaucoma, & diabetes. Tx: beta-blockers (timolol) -> laser traculoplasty as adjunct -> surgical trabeculectomy if continuous inc in IOP. How to manage eclampsia? - Correct answer 1) Administer anticonvulsant (mag sulfate). 2) Attempt delivery. What kind of eye condition is most common with neurofibromatosis type 1? - Correct answer Optic glioma (15%), mostly in kids <6yo. Hx of slowly progressive unilateral visual loss & dyschromatopsia (dec brightness sensation). Sometimes exophthalmos. How should you treat a neonatal clavicular fracture (crepitus, asymmetric Moro reflex)? - Correct answer Supportive care & avoiding tension on the affected arm. Most neonatal clavicular fractures heal within weeks. What should you be concerned about with absent Moro, biceps, & grasp reflexes in an infant? - Correct answer C5-C7 damage (ie, Erb-Duchenne paralysis), or brachial plexus injury. How do the presentations of HSV retinitis and CMV retinitis differ? - Correct answer HSV retinitis: rapidly progressing bilateral necrotizing retinitis ("acute retinal necrosis syndrome"), visual loss, keratitis, conjunctivitis with eye pain. CMV retinitis: painless, fluffy/granular retinal lesions near the retinal vessels and assoc hemorrhages, no initial conjunctivitis or keratitis. What's the leading complication of severe burns in the first week with adequate initial fluid resuscitation? - Correct answer Bacterial infection (usually bronchopneumonia or burn wound infection, from Staph aureus or Pseudomonas) -> worsening hyperglycemia (bad insulin resistance), leukocytosis, thrombocytopenia, mild hypothermia, tachypnea, & tachycardia. What specific EKG findings are associated with pericardial effusions? - Correct answer Electrical alternans (QRS complexes whose amplitudes vary from beat to beat on EKG) Tx: immediate pancreatic enzyme replacement tx, combination bronchodilator (albuterol) + chest PT. Smoking cessation of family. Many pts need G tube placement for supplemental overnight feeds. How common is infantile GERD? - Correct answer Very common - in ~50% of term infants. Sx's begin in first few months, peak ~4 mos, and generally resolve by 1 yr. Tx: upright positioning, thicken feeds, make sure quantity/timing of feeds is optimal. No PPI or imaging necessary. What hypercoagulable heritable condition is usually the cause of recurrent unexplained DVTs? - Correct answer Factor V Leiden (point mutation in gene of factor V) -> becomes resistant to inactivation by protein C. What is Legg-Calve-Perthes disease? - Correct answer Syndrome of idiopathic osteonecrosis (avascular necrosis) of the femoral head. Classically: boys age 4-10yo, chronic knee/hip pain of insidious onset, antalgic gait (limp). Etiology unclear - in some pts, may be underlying thrombophilia. Tx: maintain femoral head within acetabulum via splinting or surgery. Refrain from weight-bearing activities. Where does hematogenous osteomyelitis usually strike? - Correct answer Metaphysis of long bones (humerus, tibia, femur). Fever + systemic sx's. In which population does slipped capital femoral epiphysis usually occur? - Correct answer Obese adolescents. Mean ages: 12yo in girls, 13.5yo in boys. What condition presents with asymmetric resting tremor in the distal part of an upper extremity? - Correct answer This is the most common presenting sign of Parkinson disease. It's a clinical diagnosis (physical exam is more accurate than imaging!). What condition presents with a dull aching and fullness of the scrotum, swelling of the scrotum, negative transillumination test, and increased swelling with Valsalva maneuver? - Correct answer Varicocele - a tortuous dilation of pampiniform plexus of veins surrounding the spermatic cord & testis in the scrotum. Results from the incompetence of the venous valves, L > R (L testicular vein enters the L renal vein inferiorly at a right angle -> inc risk of impaired drainage). How to differentiate transudate from exudate fluid accumulation? - Correct answer Serum ascites albumin gradient (SAAG) = serum albumin - ascites albumin. Transudate (portal HTN, etc from inc capillary hydrostatic P): SAAG > 1.1. Exudate (infection, trauma, pancreatitis, malignancy - from inc capillary permeability): SAAG < 1.1. How would you treat hemochromatosis? - Correct answer Early dx & tx have significant impact. Tx: avoid all iron-rich foods, weekly phlebotomies for 2-3 yrs, deferoxamine (iron chelator) is 2nd line for pts who can't tolerate phlebotomies. All first-deg relatives should consider genetic testing. Labs: mildly elevated LFTs, elevated iron with >50% saturation of transferrin, elevated ferritin. What condition presents with 1-2 wks of fever, RUQ pain starting w/in 5 mos of returning from an endemic area, diarrhea, and occasionally jaundice? - Correct answer Amebic liver abscess - can also involve other organs (lung, heart, brain). Confirm dx with U/S, CT, or MRI. Next step is serologic testing for Ab's. Tx: oral metronidazole for 7-10 days. If tests are negative for ameba or pt not improving, do percutaneous drainage to r/o pyogenic absceess. *No needle aspiration b/c of risks! (bleeding, amebic peritonitis, etc) For what liver abscess would you treat with IV abx and drainage? - Correct answer Pyogenic (bacterial) abscess - typically occurs in older, debilitated pts. Use empiric broad-spectrum abx (ceftriaxone) in conjunction with metronidazole. What does an antibody titer of 1:4 mean? - Correct answer Indicates that the solution has to be diluted 4 times before the antibodies are undetectable. (Thus an Ab titer of 1:4 has less Ab's than a titer of 1:16.) *Ab titers > 1:6 means a mother is sensitized for D antigen -> RhoGAM is no longer helpful, close fetal monitoring for hemolytic dz is required. Which CN is responsible for corneal sensation? - Correct answer CN V1 How do you treat social phobia? - Correct answer Assertiveness training (a subset of CBT) + SSRI When you have a pt with hx of breast cancer, lytic bone lesions, hyperCa, and normal PTH levels, what is the underlying process? - Correct answer Breast cancer metastasizing into bone -> tumor cells secrete factors that activate osteoclasts to indirectly cause bone resorption, and produces PTHrP locally when metastasized -> hyperCa locally. Where do most (90%) medulloblastomas develop? - Correct answer In the cerebellar vermis. Posterior vermis syndrome: truncal dystaxia, horizontal nystagmus, papilledema, unbalanced gait. What condition presents with both lower and upper motor neuron damage, with muscle weakness beginning distally and asymmetrically? - Correct answer Amyotrophic lateral sclerosis (ALS). How to manage a child who has accidentally taken liquid alkali (oven cleaner)? - Correct answer 1) Maintain airway patency. 2) Upper GI endoscopy in first 24 hrs to assess injury & dictate mgmt. *Using vinegar or NG lavage are dangerous b/c they may increase the extent of the injury! What would you suspect in a 58yo man with no medical problems except recent seizures and MRI showing several lesions at the jxn of gray-white matter with edema? - Correct answer Metastatic malignancy: lung > breast (solitary met) > unknown primary > melanoma > colon (solitary met). Brain mets are the most common intracranial tumors in adults. Perform chest CT next to determine primary source. What does glioblastoma multiforme usually look like on brain imaging? - Correct answer Solitary mass with central necrosis and extensive vasogenic edema. What are the risk factors for neonatal RDS? - Correct answer Prematurity, male, C/S w/o labor, perinatal asphyxia, & maternal diabetes (fetal hyperinsulinemia counteracts actions of cortisol -> delays lung maturation). What condition presents in three phases: abd pain/n/v/diarrhea (week 1), "splinter" hemorrhages/periorbital edema (week 2), and muscle pain/tenderness/swelling/weakness? - Correct answer Trichinosis, a parasitic infection caused by the roundworm Trichinella. Acquired by eating undercooked pork containing encysted Trichinella larvae. Blood count usu shows eosinophilia. What condition presents with constipation, descending paralysis, and respiratory failure? - Correct answer Botulism What condition presents with lip & tongue swelling, associated with ACEi use? - Correct answer Angioedema. Deficiency of C1 inhibitor. What condition presents in a child <3yo who has a "barky" cough, hoarseness, and varying respiratory distress over time? - Correct answer Croup, aka laryngotracheitis or laryngotracheobronchitis. Most common agent is Parainfluenza virus. Usu a clinical dx. Lateral neck Xrays will show subglottic narrowing. How does epiglottitis usually present? - Correct answer Cough is not "croupy," and pt appears toxic - high-grade fever, tachypnea, tachycardia, stridor, & XS drooling. Lateral Xrays will show: swollen epiglottis (thumb sign), thickened aryepiglottic folds, & obliteration of vallecula. How are laryngotracheobronchitis (croup) and laryngotracheobronchopneumonitis different? - Correct answer Bacterial laryngotracheobronchopneumonitis has signs of *lower* airway involvement, like wheezing & interstitial changes on CXR. Usu w/hypoxia 2/2 lower airway dz. Croup is upper airway. How do you manage epiglottitis? - Correct answer Endotracheal intubation and set-up for possible tracheostomy (nebulized racemic epi is for croup). What deficiency would you suspect in a malnourished child with sore throat, cheilitis, stomatitis, glossitis, anemia (normocytic-normochromic), seborrheic dermatitis, & photophobia? - Correct answer Riboflavin (B1) deficiency. *Air = black! How would pulmonary infarct present on imaging? - Correct answer Pleural effusion on affected side + wedge-shaped opacity in the infarcted segment. How to treat multiple sclerosis? - Correct answer Acute exacerbation: high-dose IV steroids (methylprednisone). Mild sensory sx's: no acute tx. Long-term, in relapsing-remitting or 2ndary progressive form of MS: beta-interferon, plasmapheresis, cyclophosphamide, IVIG, & glatiramer acetate. What's the single most common cause of asymptomatic isolated elevation of alkaline phosphatase in an elderly pt? - Correct answer Paget's disease of bone (osteitis deformans) - most commonly diagnosed in asymptomatic ppl when isolated elevated alk phos is incidentally discovered on routine blood tests. What is Paget's disease of bone? - Correct answer Defective osteoid formation at sites of high bone turnover -> hypertrophy of bone. Most commonly affected sites: skull, clavicles, pelvis, & long bones. Complications include: pathologic fractures, pain, osteosarcoma, & neuro sx's. What condition presents with a virilized XX child with hx of normal internal genitalia but ambiguous external genitalia, clitoral hypertrophy, and high FSH/LH with low estrogen? - Correct answer Aromatase deficiency. Rare genetic d/o. Absence or poor fxning of enzyme converting androgens into estrogens. Later in life these pts will have delayed puberty, osteoporosis, high levels of gonadotropins and polycystic ovaries. What's Kallman's syndrome? - Correct answer Hypogonadotropic hypogonadism with anosmia (can't smell). Delayed puberty, low/absent FSH & LH levels. What're the major radiographic features of osteoarthritis? - Correct answer Predominantly affects the DIP joints (vs PIP & MCP), joint space narrowing, subchondral sclerosis, osteophytes, & subchondral cysts. What condition presents with Xray showing expansive and eccentric lytic area ("soap- bubble" appearance) in the epiphyseal region of a long bone in a young adult? - Correct answer Giant cell tumor of bone. A benign and locally aggressive skeletal neoplasm of young adults. Presentation: pain, swelling, dec range of joint motion, pathologic fractures due to thinning of bone cortex in weight-bearing areas. Most commonly distal femur & proximal tibia around knee joint. MRI: tumor w/cystic & hemorrhagic regions. Path: sheets of interspersed large osteoclast giant cells. Tx: surgery (curettage w/ or w/o bone grafting). What's a Baker's cyst? - Correct answer Popliteal synovial cyst - swelling on medial side of popliteal fossa 2/2 enlarged gastrocnemius-semimembranous bursa. Often 2ndary to degenerative joint dz or injury. What's Osgood-Schlatter dz? - Correct answer An overuse injury caused by repetitive strain. Typically seen in young kids/adolescents who have recently undergone a rapid growth spurt. X-ray of knee shows: avulsion of apophysis of tibial tubercle. What's osteitis fibrosa cystica (von Recklinghausen disease of bone)? - Correct answer Rare condition most commonly 2/2 hyperPTH from parathyroid carcinoma -> osteoclastic resorption of bone leads to replacement with fibrous tissue (brown tumors) & causes bone pain. Imaging shows: subperiosteal bone resorption, "salt-and-pepper" appearance of the skull, bone cysts, brown tumors of long bones. What condition typically causes bone pain that's worse at night and unrelated to activity, and quickly relieved by NSAIDs? - Correct answer Osteoid osteoma. Imaging shows: sclerotic cortical lesion with a central nidus of lucency. How to manage a pt with unprovoked first seizure? - Correct answer Neuro exam, urine tox screen, CT w/o contrast or MRI esp to look for head trauma (bleed). *CT w/contrast is less sensitive. Good for id'ing structural abnl or mass lesions. What is BNP secreted in response to? - Correct answer Ventricular stretch and wall tension when cardiac filling pressures are elevated -> level of BNP correlates w/severity of inc LV filling pressure (S3) & mortality. What're the risk factors for gallbladder carcinoma? - Correct answer Female, Mexican- American/Native American, age, and some of same risk factors as for chronic gallbladder inflammation (hx gallstones, porcelain gallbladder, salmonella typhi carriage, anatomic abnl, & carcinogen exposure). Most discovered incidentally. What would massive hemoptysis be defined as? - Correct answer >600 mL expectorated blood over a 24-hr period, or a bleeding rate of >100 ml/hr. *Greatest danger is asphyxiation due to airway flooding. How to manage massive hemoptysis? - Correct answer Establish airway - place bleeding lung in dependent position to avoid blood collection in opposite lung. Bronchoscopy is initial procedure of choice - can localize bleeding site, provide suctioning, & include other tx. What's adenomyosis? - Correct answer Presence of endometrial glands in the uterine muscle, most commonly in women >40yo, typically presents with 2ndary dysmenorrhea & menorrhagia. What are some ototoxic medications (cause sensorineural hearing loss)? - Correct answer Vancomycin, loop diuretics (furosemide), aminoglycoside abx, chemo agents (doxorubicin), & aspirin. What are some predisposing factors for UTIs? - Correct answer Female (shorter urethra), recent abx, sexual intercourse, diaphragm or spermicide use, family hx of multiple UTIs. Where are the two watershed zones in the colon? - Correct answer 1) Splenic flexure (supplied by narrow terminal branches of SMA), and 2) Recto-sigmoid junction (supplied by narrow terminal branches of IMA). Most vulnerable to ischemia during systemic hypotension. What would you suspect in a patient with evidence of atherosclerotic vascular disease, present with abd pain followed by bloody diarrhea, and have minimal abd exam findings? - Correct answer Ischemic colitis What should you suspect in a child with HSP and presents with abdominal pain? - Correct answer 1) GI bleed, and 2) Intussusception. *Volvulus is a condition of the elderly. How should you manage a boy with a febrile illness and 2+ proteinuria on urine dipstick? - Correct answer Proteinuria in children can be transient, orthostatic, or persistent. Transient is the most common, and can be caused by fever, exercise, SZ, stress, or volume depletion. Orthostatic is very common in adolescent boys. Both are benign. If UA is otherwise normal, urine dipstick should be repeated on 2+ additional specimens -> if positive, refer to peds nephrologist for underlying renal dz -> 24-hr urine collection for protein, renal U/S, +/- renal biopsy. What is the most common cause of pleural effusion? - Correct answer CHF What are serum characteristics of a transudate (like a pleural effusion from CHF)? - Correct answer 1) Pleural/serum protein ratio <0.5. 2) Pleural/serum LDH <0.6. 3) Pleural LDH < 2/3 of upper limit of normal serum LDH. *Pleural pH<7.3 indicates pleural inflammation. (pH<7.2 requires a chest tube aspiration to prevent empyema.) What 2/3 signs do >80% of aortic dissections have? - Correct answer 1) Abrupt onset of "tearing" chest/back pain. 2) Variation in pulse or bp btw right and left arms. 3) Widened mediastinum on CXR. *Dissection of ascending aorta may extend into pericardium (cardiac tamponade), coronary arteries (MI), or carotid arteries (stroke). What are the teratogenic vs recommended anti-hypertensives during pregnancy? - Correct answer Teratogenic: ACEi, ARBs. Recommended: Labetalol, Methyldopa. *If a pregnant pt presents with essential HTN but isn't on anti-hypertensives, only start tx if bp >150/95. What's spondylolisthesis? - Correct answer Developmental d/o characterized by a forward slip of vertebrae (usually L5 over S1) -> chronic back pain and neuro dysfxn if significant, palpable "step-off" of vertebral displacement is detected if problem is severe. What's another name for Wilson's disease? - Correct answer Hepatolenticular degeneration What are the indications for the evaluation of neonatal jaundice? - Correct answer 1) Jaundice appearing in first 24-36 hrs of life, 2) Bilirubin rising faster than 5 mg/dl/24hrs, 3) bilirubin >12 in full-term or 10-14 in preterm infants, 4) Jaundice persisting after 10-14 days of life, 5) Conjugated hyperbili >2, and 6) presence of signs or sx's. What to do next with a pregnant woman who's had a nonreassuring non-stress test and vibroacoustic stimulation has failed to increase variability? - Correct answer Do either a biophysical profile or a contraction stress test. What does a biophysical profile measure? - Correct answer 5 parameters: 1) non-stress test, 2) amniotic fluid volume, 3) gross fetal mvmts, 4) extremity tone, and 5) fetal breathing. (Last 4 assessed with OB U/S.) What's a major advantage of chorionic villus sampling over amniocentesis? And major disadvantage? - Correct answer Advantage: it can be performed earlier: 10-12 wks vs 15-20 wks. Disadvantage: fewer physicians trained, may be associated with transverse digital deficiency (limb defect). How long does it take to clear beta-hCG levels after an abortion? - Correct answer Several weeks What's levonorgestrel? - Correct answer Emergency contraception within 72 hrs of unprotected intercourse What are some causes of arrest of labor during pregnancy? - Correct answer Contractions may not be adequate, fetus may have a malpresentation, or maternal pelvis may not be able to accommodate the fetus What's the most common cause of urinary incontinence? - Correct answer Stress incontinence, caused by pelvic floor weakness -> urine loss from increased intra- abdominal pressure (coughing, sneezing). Pelvic exam usually reveals a cystocele. What's the classic triad of dysuria, dyspareunia, and postvoid dribbling indicative of? - Correct answer Urethral diverticulum (outpouching of tissue from urethra into urethrovaginal space). What're common causes of denervation to the bladder, leading to hypotonic bladder & urinary incontinence? - Correct answer Diabetes and multiple sclerosis What's a contraindication to a vaginal birth after cesarean (VBAC)? - Correct answer Classic uterine scar from previous C/S (vertical incision) -> 10% risk of uterine rupture What's the most common sx of ovarian cancer in pts? - Correct answer Increased abdominal girth caused by ascites. Other sx's: fatigue, urinary frequency, SOB. Why is estrogen contraindicated in breastfeeding women and/or in first 3 weeks after delivery? - Correct answer Because of effects on breastfeeding and on the infant. Also, first 3 wks after delivery is considered to be a possibly hypercoagulable time (estrogen increases thrombus risk). What's the major risk factor for postpartum endometritis? - Correct answer C/S What's the folic acid supplementation recommendation for women wanting to become pregnant? - Correct answer 0.4 mg/day. For women who've previously had a neural tube defect-affected pregnancy: 4 mg/day, continuing through the 1st trimester. Why is a vaginal exam contraindicated without an U/S first, in vaginal bleeding during pregnancy? - Correct answer Examiner's finger can acutely worsen the hemorrhage and induce further bleeding. How is emancipated minor defined? - Correct answer Married, in the military, or lives separately from parents & manages own finances How does steroid acne present differently from adolescent acne? - Correct answer Steroid acne: monomorphous pink papules & absence of comedones. Adolescent: open and closed comedones and inflammatory nodules in differing stages of evolution. What are the most common secondary malignancies affecting patients previously treated for Hodgkin's disease (esp with chemo + radiation)? - Correct answer Lung and breast cancers What's the underlying mechanism behind the development of cholesterol gallstones in pregnancy and in women taking OCPs? - Correct answer Estrogen-induced increase in cholesterol secretion (and progesterone causing reduction in bile acid secretion -> increased cholesterol saturation of bile) How can chemotherapy lead to bacteremia directly? - Correct answer Chemo -> disruption of skin and mucosal barrier -> seeding of bacteria into the bloodstream. Most commonly the GI tract, and gram-negatives, particularly Pseudomonas. What condition presents with severe symptomatic HTN with HA, epistaxis, and evidence of LVH in a young person? - Correct answer Coarctation of the aorta What're the 4 T's of the differential for an anterior mediastinal mass? - Correct answer Thymoma, Teratoma (& other germ cell tumors), Thyroid neoplasm, & Terrible lymphoma What's the most common paroxysmal tachycardia in people without structural heart disease? - Correct answer Paroxysmal supraventricular tachycardia (starts in atria or AV node) What kind of immune response does the polyvalent pneumococcal vaccine (not conjugated with protein) elicit? - Correct answer Polyvalent pneumococcal vaccine not conjugated with protein contains capsular polysaccharides of the 23 most prevalent types of pneumococcus. Polysaccharides can't be presented to T-cells. So this vaccines yields a T-cell-independent B-cell response. Which cells do natural killer (NK) cells attack? - Correct answer Primarily cancer cells & virus-infected cells. Don't require signals from other immune cells for activation. Which vaccines induce a predominantly IgA response? - Correct answer Oral polio vaccine, which promotes the development of anti-poliovirus IgA in the GI tract. What should you use when giving IV boluses (ie, initial resuscitation)? - Correct answer Only isotonic solutions such as normal saline (0.9% saline) or lactated Ringer's How does bowel ischemia arise from AAA repair? - Correct answer Inadequate colonic collateral arterial perfusion to the L & sigmoid colon after loss of the inferior mesenteric artery during aortic graft replacement. How would an iatrogenic bowel perforation present? - Correct answer Always suspect it after any abdominal operation - you'd see signs of peritoneal irritation How to differentiate a hydrocele from other testicular masses? - Correct answer Hydrocele will transilluminate. Most will resolve spontaneously by 1yo. Beyond 1yo, there's inc risk of indirect inguinal hernia -> need surgery. In which phase is the cervical mucus profuse, clear, and thin? - Correct answer Ovulatory phase (pH is 6.5+ and will demonstrate "ferning"). In other phases, it's scant, opaque, and thick. What cardiac conditions are Marfan's pts at higher risk of? - Correct answer Aortic dissection and mitral valve prolapse What's the relationship btw hypothyroidism and hyperprolactinemia? - Correct answer TRH stimulates prolactin production. (Prolactin is inhibited by dopamine and stimulated by serotonin and TRH.) Hypothyroidism -> increased TRH/TSH -> increased prolactin. What conditions is pyoderma gangrenosum associated with? - Correct answer IBD, rheumatoid arthritis, leukemia, & chronic active hepatitis. Inflammatory ulcer with undermined borders, may occur anywhere on body. What are some CXR findings that are indicative of blunt aortic injury (usu a mechanism of rapid deceleration)? - Correct answer Wide mediastinum (>6cm upright), obscured aortic knob, left "apical cap" (pleural blood above L lung), large L hemothorax, deviation of NG tube rightward, deviation of trachea rightward, & wide L paravertebral stripe. What's a keratoacanthoma? - Correct answer A rapidly growing keratotic lesion often described as "crateriform" or "volcano-like" - many regarded as well-differentiated forms of squamous cell carcinoma -> excised. What's a pyogenic granuloma? - Correct answer Rapidly growing vascular lesion most often occurring at sites of minor trauma. Resemble granulation tissue on histology. Red papules with collarette of scale. What's the most common cutaneous malignancy? - Correct answer Basal cell carcinoma How to best evaluate sx's of acute brain stem infarction (and CT showing such)? - Correct answer Angiography of cerebral/neck vessels will evaluate sources of bleeding/thrombus/stenosis. How to manage an actively seizing patient who has been noncompliant with his AED, phenytoin? - Correct answer 1) Lorazepam in increasing doses -> 2) IV fosphenytoin vs. propofol vs. barbiturate -> 3) Move pt to OR and use volatile gases (isoflorane). What's the target systolic bp in the case of a subarachnoid hemorrhage? - Correct answer 160-170 mmHg systolic; use IV nitroprusside (can be titrated easily) Which drugs can cause phototoxic drug eruption (interaction of UV light with drug -> ROS)? - Correct answer Tetracyclines (esp doxycycline), thiazides, sulfonamides, fluoroquinolones, & NSAIDs What's polymorphous light eruption? - Correct answer Idiopathic immune-mediated rxn to UV exposure: confluent pruritic papules & plaques in a photodistributed pattern. What condition presents with severe pain and blurry vision in one eye, brought on by spending a long period of time in a dark room or taking an anticholinergic medication? - Correct answer Acute closed-angle glaucoma, an emergency brought on by increased intraocular pressure. *Hazy-appearing cornea with a partially dilated & fixed pupil. *Open-angle glaucoma is >90% of glaucoma cases, and is insidious over time. What are some complications of PEEP ventilation? - Correct answer Barotraumas (resulting in pneumothorax, pneumomediastinum, subcutaneous emphysema) and ventilator-associated lung injury (2/2 overdistension of alveoli). Inc intrathoracic P -> dec venous return (preload) -> dec cardiac output -> systemic hypotension. How to manage emergency burn patients? - Correct answer 1) ABCs (low threshold for intubation, like hoarseness) -> 2) Assess % of BSA burned and give IVF (large initial bolus of Ringer's lactate 1L/h) -> 3) Insert Foley to monitor fluid balance -> 4) Cover burns with Vaseline gauze to prevent fluid evaporation -> 5) Tests like bronchoscopy & CXR. What's the cause of pronator drift? - Correct answer Upper motor neuron weakness: supination is weaker than pronation in the upper extremity. What germline mutation does familial melanoma syndrome arise from? - Correct answer CDKN2A (cyclin-dependent kinase inhibitor 2A). Also assoc w/pancreatic cancer. What's the constellation of cancers associated with Li-Fraumeni syndrome (loss of p53 function)? - Correct answer Breast carcinoma, soft-tissue sarcoma, osteosarcoma, brain tumors, adrenocortical carcinoma, & Wilms tumor. Which cancers is the RET proto-oncogene mutated in? - Correct answer MEN types 2A & 2B. Which cancer are TSC1 & TSC2 mutated in? - Correct answer Tuberous sclerosis. They code for proteins hamartin & tuberin, respectively. Tuberous sclerosis: facial angiofibromas, periungual fibromas, hypopigmented "ash leaf" macules, and CT nevus (Shagreen patch). Seizures and commonly involvement of heart, lungs, & kidneys. What rapidly counteracts hyperK with ECG changes, and should be administered first in this emergency? - Correct answer IV calcium gluconate - stabilizes cardiac membranes. Then use other meds to decrease the K. What's the lifetime risk of developing bipolar d/o for the general population, versus someone with a first degree relative with bipolar d/o? - Correct answer General population: 1% 1st-degree relative: 5-10% When should you attempt to convert a breech presentation (vertex at fundus) into vertex presentation? - Correct answer Not before the 37th wk GA, b/c most will spontaneously convert by then. After 37th wk, attempt external cephalic version, assuming no contraindications (placental abnl, fetopelvic disproportion, hyperextended head). What are the 2 most common causes of hypopituitarism following pregnancy (postpartum period)? - Correct answer Sheehan's syndrome (ischemic necrosis of pituitary, sometimes even hypothalamus, b/c of peripartum bleeding) & lymphocytic hypophysitis. What condition presents with pain of the eye, photophobia, decreased vision, and dendritic ulcer? - Correct answer Herpes simplex keratitis (as differentiated from herpes zoster ophthalmicus - which is shingles from trigeminal ganglion). What's a well-known side effect of thiazide diuretic therapy? - Correct answer Dec glucose tolerance -> hyperglycemia. Thiazides can also affect lipid metabolism -> inc LDL & triglycerides. Electrolyte abnormalities: hypoNa, hypoK, hyperCa What should you suspect in an ill-appearing, jaundiced infant between 3-7 days of life? - Correct answer Neonatal sepsis. Dx: CBC, UA, urine/blood cultures, LP. Where does most of diffuse axonal injury (traumatic brain injuries) occur? - Correct answer Gray-white matter junction, where the density difference is the maximum. Dx: MRI is more sensitive than CT. What would you suspect in vaginal discharge/bleeding in female infants <3 mos old? - Correct answer The effects of maternal estrogens (can cross the placenta) - disappears when hormone is cleared from circulation. How can you normalize high homocysteine levels? - Correct answer Give pyridoxine (B6) and folate. Add B12 if there's a deficiency. Unclear whether this also reverses hypercoagulability. What stains positive with Prussian blue? - Correct answer Hemosiderin (iron) When should you biopsy a lymph node? - Correct answer >2 cm diameter (associated with malignancy or granulomatous disease). <1 cm almost always from benign process. How to differentiate between asthma and COPD? - Correct answer Asthma: significant improvement in FEV1 after bronchodilator administration. COPD: low diffusion capacity (DLCO), characteristic chest CT. What kinds of cancer does tamoxifen INCREASE the risk of? - Correct answer Uterine cancers - endometrial cancer (lining of uterus) and uterine sarcoma (muscular wall of uterus). *Tamoxifen has a mixed agonist & antagonist activity on estrogen receptors. Inc risk of venous thrombosis too. What can cause hypoxemia? - Correct answer Reduced inspired O2 tension, hypoventilation, diffusion limitation, shunt, & V/Q mismatch. *Hypoventilation associated with normal A-a gradient & respiratory acidosis. (V/Q mismatch with respiratory alkalosis because of compensatory tachypnea.) What infection are patients with hemochromatosis and cirrhosis at increased risk for? - Correct answer Listeria monocytogenes (inc bacterial virulence in presence fo high serum iron, impaired phagocytosis 2/2 iron overload in reticuloendothelial cells). Also at risk for Yersinia enterocolitica and septicemia from Vibrio vulnificus (iron-loving bacteria). What is an abdominal CT good for detecting? - Correct answer Bile & pancreatic duct dilation, mass lesions in pancreas, & indications of extrahepatic spread (mets or ascites). What's flumazenil used for? - Correct answer Treats BDZ toxicity. What condition presents with hx recurrent UTI secondary to reflux, and IV pyelography showing focal parenchymal scarring & blunting of calyces? - Correct answer Chronic pyelonephritis. What condition presents with postprandial abdominal cramps, weakness, light- headedness, and diaphoresis after having had a partial gastrectomy? - Correct answer Early dumping syndrome, a common postgastrectomy complication: rapid emptying of hypertonic gastric content into the duodenum & small intestine -> fluid shift from intravascular space to small intestine, release of intestinal vasoactive polypeptides, & stimulation of autonomic reflexes. *Clinical dx. Tx: dietary modification (small frequent meals, avoid simple carbs) -> octreotide or surgery in refractory cases. What're the most common sites of mets? - Correct answer Lymph nodes, then liver (large size, dual blood supply, high perfusion rate, & filtration function of Kupffer cells). What condition can present with nephrotic syndrome (facial swelling, leg edema, massive proteinuria) with palpable kidneys, hepatomegaly, and ventricular hypertrophy in the setting of chronic inflammatory dz (psoriasis)? - Correct answer Secondary amyloidosis as a unifying diagnosis. Tx underlying inflammatory disease, and colchicine for secondary amyloidosis. What's the most common organism responsible for pericarditis? - Correct answer Coxsackie virus. Coxsackie B also causes viral myocarditis. What does pulmonary capillary wedge pressure mean? - Correct answer It indicates preload. -> Normal/low in septic shock (peripheral vasodilation, decreased systemic vascular resistance). High in cardiogenic shock (heart can't pump). Low in hypovolemic shock (not enough fluid). What are common clinical findings of septic shock? - Correct answer Hypotension, warm -> cool extremities, elevated lactate levels. Septic shock = form of distributive shock due to underlying systemic infection involving circulatory system. What's the most common congenital cause of aplastic anemia? - Correct answer Fanconi anemia. AR or X-linked. Defect in DNA repair genes -> dx: chromosomal breaks on genetic analysis. Thrombocytopenia, fatigue (macrocytic anemia). *Tx: hematopoietic stem cell transplant. What's the mechanism of iron poisoning? - Correct answer Free radical production and lipid peroxidation -> impairs cell processes -> systemic manifestations (abd pain, hematemesis, hypovolemic shock, & metabolic acidosis). What is sodium bicarbonate the antidote for? - Correct answer TCA & aspirin overdose. How to treat severe lithium toxicity? - Correct answer Hemodialysis - lithium is the most dialyzable toxin. What injury commonly follows blunt abdominal trauma, but can be missed on initial abdominal CT? - Correct answer Pancreatic injury (contusion, crush injury, laceration, or transection, when pancreas is compressed against the vertebral column). What patient group is spironolactone most effective with treating? - Correct answer Patients with ascites from cirrhosis, and those with heart failure class III & IV. How should you treat patients with acne scars? - Correct answer Oral isotretinoin (isotretinoin is very effective for treating nodulocystic acne). Can also give to pts with moderate-to-severe acne that's predominantly nodulocystic. What's demeclocycline? - Correct answer Inhibits effect of AVP or ADH on the distal convoluted tubule in the kidneys. May be used to tx SIADH (but worsens diabetes insipidus). What does blood at the beginning of urination tell you about where the bleeding is originating? - Correct answer Blood at the beginning: urethral lesion. Blood at the end: prostate or bladder cause. Blood during the entire urinary cycle (WITHOUT clots): disease in ureters or kidneys. What viruses cause almost 90% of cases of viral meningitis? - Correct answer Non- polio enteroviruses, such as echovirus and coxsackievirus. What imaging can you use to identify uric acid stones? - Correct answer CT or IV pyelography, because uric acid stones are radiolucent (Xray can't see). What's Jarisch-Herxheimer reaction? - Correct answer May develop in syphilis treatment - spirochetes die rapidly -> release of antigen-Ab complexes in blood -> immunologic rxn (seems like an acute flare-up of syphilis). What condition presents with a pigmented scaly rash on face/neck/back of hands that increases on sunlight exposure, diarrhea, and irritability/depressed mood? - Correct answer Pellagra (niacin deficiency). Ppl eating corn-based diets (India, Africa, China), alcoholics, carcinoid syndrome, & Hartnup's dz. 3 D's: Diarrhea, Dermatitis (in sun- exposed areas), & Dementia. Death if untreated. When should primitive reflexes start disappearing in infants? - Correct answer By age 4 months, Moro and grasp reflexes should be disappearing. Tongue protrusion reflex should also disappear to allow for coordination to start ingesting solid foods. When should the Babinski reflex disappear? - Correct answer Sometime between 1-2 yo. What condition presents with sudden loss of vision in one eye and floaters in the visual field, and on fundoscopy, a fundus that's hard to visualize with obscure details? - Correct answer Vitreous hemorrhage, most commonly caused by diabetic retinopathy. What should you suspect in an infant who presents with cyanosis when feeding but relieved by crying? - Correct answer Choanal atresia. Most common nasal malformation. May be isolated or part of a dysmorphic syndrome. Can't insert a nasal catheter. Dx: CT with intranasal contrast shows narrowing at level of the pterygoid plate. What are female offspring of women who used DES during pregnancy more at risk of? - Correct answer Clear cell adenocarcinoma of the vagina & cervix, as well as cervical anomalies & uterine malformations. *Exposed males are at risk of cryptorchidism, microphallus, hypospadias, & testicular hypoplasia. What are risk factors for endometrial adenocarcinoma? - Correct answer Obesity, nulliparity, late menopause, HTN, DM, chronic unopposed estrogen stimulation, & chronic tamoxifen use. What are risk factors for ovarian cancer? - Correct answer Family hx, nulliparity, & lack of prior OCP use. What condition presents with normal skin at birth, with gradual progression to dry scaly skin? - Correct answer Ichthyosis vulgaris. Hereditary or acquired. Dry & rough skin with horny plates over extensor surfaces of limbs. Worsens in winter. Aka "lizard skin." How to manage a transplant patient (in terms of prophylaxis against infection)? - Correct answer Give oral TMP-SMX, ganciclovir (prevent CMV), and flu, pneumococcus, and hep B vaccines. What're the most common causes of neonatal conjunctivitis? - Correct answer Chlamydia, gonorrhea, & chemicals. Tx of chlamydial conjunctivitis or PNA = 14 days of oral erythromycin (though risk of hypertrophic pyloric stenosis). Prophylaxis = screening & tx of mother before delivery. Prophylaxis for gonococcal conjunctivitis = erythromycin eye drops at birth. What's the most common coagulopathy in patients with malignancies (esp gastric, breast, & lung)? - Correct answer DIC (disseminated intravascular coagulation). Labs: thrombocytopenia, dec fibrinogen, inc INR. Microangiopathic hemolytic anemia is commonly seen. What's the initial tx for acute myocardial ischemia? - Correct answer Morphine, O2, nitrates, aspirin, beta-blockers, LMW heparin, & possibly statins. When do you start medical tx for gout? - Correct answer Only indicated in pts with repeated attacks of gout, tophi suggestive of chronic disease, or renal insufficiency. Try lifestyle changes first (ex: EtOH cessation). What's the difference between factitious disorder and malingering? - Correct answer Factitious d/o is consciously making yourself sick to assume the sick role. In malingering, you have a secondary gain for getting sick (disability, avoid jail, lawsuit, or narcotics). Which HIV drug can cause insomnia with vivid or bizarre dreams, and CNS side effects? - Correct answer Efavirenz (non-nucleoside reverse transcriptase inhibitor). Side effects usually occur with the first few doses and subside over time. Which HIV drug can cause pancreatitis? - Correct answer Didanosine. What's the treatment for any abscess? - Correct answer Incision & drainage. Pinpoint fixed pupils suggest what drug? - Correct answer Opioid overdose. (Cocaine would be dilated pupils.) How can those with hypothyroidism develop carpal tunnel syndrome? - Correct answer Accumulation of matrix substances (mucin, mucopolysaccharide) within both the perineurium of the median nerve as well as the tendons passing through the carpal tunnel. Who can get carpal tunnel syndrome? - Correct answer Besides just an overuse d/o, carpal tunnel syndrome can arise in: pregnancy, hypothyroidism, amyloidosis (deposition of beta 2 microglobulin), rheumatoid arthritis (tenosynovial inflammation), and acromegaly (synovial tendon hyperplasia). What causes central cyanosis, versus peripheral cyanosis? - Correct answer Central cyanosis is caused by low arterial O2 saturation, while peripheral cyanosis is caused by increased O2 extraction 2/2 sluggish blood flow. What condition presents with yellowish streaks on the palms, and severe epigastric pain & n/v after drinking alcohol? - Correct answer Familial dysbetalipoproteinemia -> severe hypertriglyceridemia. Alcohol consumption can case recurrent pancreatitis. Tx: fenofibrate. How do OCPs decrease the risk of ovarian cancer? - Correct answer Reduces the number of lifetime ovulations. Combination OCPs are also associated with a decreased risk of endometrial cancer, but a slightly increased risk of cervical and breast cancer. What's the mnemonic for hyperCa? - Correct answer Bones, stones, groans, & psych overtones. Meaning: bone pain, nephrolithiasis, GI sx's, & neuro sx's. How to manage acute prostatitis? - Correct answer Fluoroquinolone (ofloxacin) for 4-6 wks (long-term tx required for prostatitis). Most often caused by E. coli or Chlamydia. Blood accumulating in which body parts can lead to hypovolemic shock? - Correct answer Thorax, abdomen, pelvis, and femur (fracture). What're the most likely causes of postop fever and when do they occur? - Correct answer Postop day 1: atelectasis. Day 3: UTI. Day 5: DVT. Day 7: wound infection. Days 10-15: deep abscesses. What are risk factors for squamous cell carcinoma of the head & neck? - Correct answer Alcoholism, chronic tobacco use, and infection with HPV (esp types 16, 18, & 30). Erythroplakia & leukoplakia are common precursor lesions. What diagnostic studies should you do with GI bleeding, depending on bleeding rate? - Correct answer Bleeding >2 mL/min: angiogram. Bleeding <0.5 mL/min: wait until bleeding stops and then do a colonoscopy. Bleeding between 0.5-2 mL/min: tagged RBC study. What's the Weber tuning fork test? - Correct answer Tuning fork struck and placed at midline on the skull. A pt with sensorineural hearing loss will hear it louder in the uninjured ear. A pt with conductive hearing loss will hear it louder in the injured ear (background noise decreased). What should you evaluate before starting lithium on a young woman? - Correct answer Thyroid function (can give hypothyroidism), renal function (creatinine), and check for pregnancy. What's the most common cause of XS postpartum blood loss/hemorrhage? - Correct answer Uterine atony. Initial tx: bimanual uterine massage, fluid resuscitation, uterotonic agents (oxytocin, methylergonovine, carboprost), and blood transfusion as needed. What's the most common cause of hemoptysis in adults with a hx of smoking? - Correct answer Pulmonary airway disease (chronic bronchitis, bronchogenic carcinoma, bronchiectasis). Which patient populations are more prone to getting Staph aureus PNA? - Correct answer Hospitalized pts, nursing home residents, IV drug users, pts with cystic fibrosis, & people with recent influenza infection. How do allergic conjunctivitis and atopic keratoconjunctivitis differ symptomatically? - Correct answer Atopic keratoconjunctivitis is a severe form of ocular allergy. Sx: itching, tearing, thick mucus discharge, photophobia, and blurred vision. Usually allergic conjunctivitis doesn't have visual disturbances. What's dacryocystitis? - Correct answer Infection of the lacrimal sac 2/2 obstruction of nasolacrimal duct. Pain, swelling, tenderness, & redness in tear sac area. +/- mucus or pus expressed. What's the presentation of pontine hemorrhage? - Correct answer Complete paraplegia -> deep coma within a few minutes. Pinpoint pupils that are reactive to light. Decerebrate rigidity. What's the most common site of hypertensive hemorrhages? - Correct answer Putamen. The internal capsule that lies adjacent to the putamen is almost always involved -> contralateral dense hemiparesis (pure motor stroke). How does acyclovir damage the kidneys? - Correct answer Poorly soluble in urine -> precipitates in renal tubules, causing obstruction and acute renal failure. Crystalluria with renal tubular obstruction in large parenteral doses of acyclovir. What are the TORCHES infections, and what sx's are they characterized by? - Correct answer Toxo, rubella, CMV, HSV, HIV, syphilis. Sx's: hepatosplenomegaly, deafness, microcephaly, growth retardation, chorioretinitis, & thrombocytopenia. *Congenital rubella: sensorineural hearing loss, "blueberry muffin" purpura, microcephaly. How do the presentations of edema from hypoalbuminemia and primary glomerular damage (glomerulonephritis) differ? - Correct answer Hypoalbuminemia can cause significant peripheral edema but does NOT usually cause pulmonary edema! Alveolar capillaries have a higher permeability to albumin at baseline (reducing oncotic pressure difference) & greater lymphatic flow than skeletal muscles, protecting the lungs from edema in hypoalbuminemia. Whereas glomerulonephritis can give edema all over. What's the threshold of severe preeclampsia? - Correct answer HTN > 160/110 mmHg, proteinuria > 5g/24hr, oliguria, elevated liver enzymes, thrombocytopenia, & possibly pulm edema. What condition presents with widened mediastinum on CXR and small amount of pericardial fluid on echo, 3 days after cardiac surgery? - Correct answer Acute mediastinitis, usually 2/2 intraop wound contamination. Tx: drainage, surgical debridement with closure, and prolonged abx.
Docsity logo



Copyright © 2024 Ladybird Srl - Via Leonardo da Vinci 16, 10126, Torino, Italy - VAT 10816460017 - All rights reserved